Download as pdf or txt
Download as pdf or txt
You are on page 1of 45

c) Most vitamin D in the body is not

PEDIATRICS obtained from food, so access to food is not


an important factor in determining
1. Deficiency of vitamin A in children vitamin D status
causes: d) Diets containing sufficient amounts of
a) Goiter other nutrients to promote growth may
b) Poor cognitive development increase vitamin D requirements
c) Poor bone growth to the point of becoming limiting.
d) Increased risk of mortality e) Foods containing large amounts of
с) Beri-beri vitamin D are eaten by rich people for
various cultural reasons
2. Severe acute malnutrition in young
children is defined as: 6. The mother of the 4-month's child has
a) Weight-for-age Z score <-3 addressed to the doctor with complaints on a
b) Height-for-age Z score <-3 and edema decrease of appetite,
c) Height-for-age Z score <-3 or weight-for- regurgitation, subferile temperature in her
height Z score <-3 or edema child. It is known, that from the 2nd month
d) Height-for-age Z score <-3 or weight-for- of life the child receives vitamin
age Z score <-3 or edema D3 in preventive dose /500 IU each day/,
e) Weight-for-age Z score <+3 from the 3-rd month of life because of
irritability and sweating was prescribed
3. Deficiency of which of the following general ultra-violet insolation. Objectively:
nutrients can lead to anemia? large fontanelle is closed, skin with perioral
a) Iodine and vitamin C cyanosis. Laboratory
b) Copper and iron investigations: Sulcovich test is positive
c) Zinc and protein (++), serum calcium – 3.5 mmol/l. Name the
d) Vitamin D and zinc most probable diagnosis:
e) Vitamin C and iron a) Hypervitaminosis D
b) Spasmophylia
4. For which groups of people does the c) Personal intolerance of vitamin D
World Health Organization recommend zinc d) Hypovitaminosis D
supplements? e) Avitaminosis D
a) Pregnant and lactating women
b) Children with severe malnutrition or 7. Which features are important in the
diarrhea differential diagnosis of biliary tract
c) All children in low-income areas with a dyskinesia and cholecystitis?
high prevalence of stunting a) Pain in the right upper quadrant
d) Older people with low incomes b) Symptoms vascular dystonia
e) Children with anemia c) Disease duration is 2 years
d) Enlargement of the liver
5. Vitamin D deficiency can often be found e) Enlargement of the liver and spleen
as a deficiency of a single nutrient, that is, 8. A girl, 12 years, complains of paroxysmal
in a well-nourished person. pain in the right upper quadrant, which
The main reason why this may happen is the increases during exercise, after
following: eating fatty foods, nausea, loss of appetite,
a) Foods containing high amounts of headache, fatigue. Sick for 5 years. These
vitamin D are eaten by both poor and rich objective examination: skin pale
people depending on various cultural tongue with yellow-gray stratification.
reasons Abdomen soft, painful in the right upper
b) The vitamin D content of foods depends quadrant, positive symptoms Ortner,
on the soil in which the food was grown, so Murphy, Kera, the liver acts from the edge
is not related to wealth or of costal arch to 2 cm edge sharp.The
age tendency to constipation.These
ultrasound: thickened wall and sealed d) Clarithromycin + amoxicillin + trihopol
intrahepatic bile ducts and gall bladder. e) Clarithromycin + amoxicillin + de nol
What is the main diagnosis you set sick?
a) Acute cholecystitis. 13. Infants with PI (ponderal index) of 0.9 –
b) Chronic cholecystocholangitis 1.1 and NI (nutritional index) of 1.0 – 0.9
c) Biliary dyskinesia hypokinetic type are considered:
d) Biliary dyskinesia hypertensive type a) Atrophic
e) Chronic cholecystitis9. Identify the most b) Eutrophic
important sign of B12-deficiency anemia. c) Hypertrophic
a) Hyperchromic anemia d) Hypotrophic
b) Hypochromic anemia e) Hypotrophic and hypertrophic
c) Thrombocytopenia
d) Anemia, lymphopenia, monocytopenia 14. A positive Murphy's sign is highly
e) Leukemia indicative of acute cholecystitis. Which of
the following statements
10. The mother complains of lack of teeth demonstrates this maneuver?
and lower limb deformation in a child 8 1.
months. The baby was born Pain during inhalation as the examiner
weighing 3000 grams healthy from birth on places his/her hand over the rt. upper
artificial feeding, prevention of rickets quadrant
received. During the inspection, 2.
the overall condition is not violated, pale Pain during exhalation as the examiner
skin, teeth no large fontanel has a size of 3 x places his/her hand over the rt. upper
3 cm lower aperture chest quadrant
expanded palpable ribs rosary. There O- 3.
curvature of the lower extremities. With the Pain in rt. lower quadrant on palpation of
weakening of the heart listen the lt. lower quadrant
colors, functional systolic murmur. What is 4.
your diagnosis? Pain during hyper-extension of the rt. hip
a) Rickets joint
b) Congenital brittle bones 5.
c) Hondrodystrofiya Pain on palpation over Burney's point
d) The vitamin D-dependent rickets
e) Tubulopathy 15. In the children's department taken the
girl under the age of 8 years. He complains
11. Forms of chronic nutritional disorders in of frequent attacks of pain in
children are: the right upper quadrant and near the belly
a) Hypotrophy, Hypostature, Paratrophy button that occurs after physical activity
b) Dystrophy, Hypostature, Paratrophy over the last 3 months, tendency
c) Dystrophy, Hypostature, hypertrophy to constipation. During the physical
d) Hypotrophy, Hypostature, hypertrophy examination, skin color natural, soft
e) Dystrophy, Hypostature, hyportrophy abdomen, pain in the right upper quadrant,
the liver acts from the edge of costal arch to
12. Boy, 14 years old, 3 years periodically 2 cm elastic. When duodenal sounding
complains of pain in the epigastric region on portions could not be obtained, a
an empty stomach, nausea portion of A - unchanged. Worsening of the
and heartburn. During disease is most likely the patient?
fibrogastroduodenoscopy gastroduodenitis a) Chronic cholecystocholangitis.
and the signs of ulcerative mucosal defect b) Biliary dyskinesia hypertensive type.
duodenum. What eradication complex c) Biliary dyskinesia hypokinetic-hypertonic
medications most effective in the treatment by type.
of peptic ulcer? d) Biliary dyskinesia on-hypertensive
a) Tetracycline + + trihopol hiperkinetych-it type
b) Amoxicillin + + trihopol + thiamin e) Chronic cholecystitis
c) Trihopol+ de nol + + papaverine.
16.Iron deficiency anemia is characterized b) Infiltrative shadows
by: c) Emphysema
a) Low serum glucose d) Dilation of lungs' roots
b) Low serum ferritin e) Thickening and deformation of the
c) High level of enzymes bronchovascular pattern of the lungs
d) Low level of enzymes
e) Low serum Iron 23.In most cases an acute pneumonia at
children of the early age develops as a result
17. Hospital pneumonia is one that of:
developed: a) Overheat
a) during 6 hours being in the hospital b) Super cooling
b) during 12 hours being in the hospital c) Violation of the regime
c) during a 24 hours being in the hospital d) Upper respiratory tract viral infection
d) during 48 hours being in the hospital e) Upper respiratory tract bacterial infection
e) during 72 hours being in the hospital
24. To the criteria of an acute pneumonia
18. A newborn baby presented to the diagnosis belong:
emergency with complaint of vomiting a) Lymphocytes in complete blood count
since birth. X-ray abdomen revealed b) Epidemiological history
double bubble. The diagnosis is: c) Changes in nasal ways
a) Duodenal atresia d) Respiratory acidosis
b) Esophageal atresia e) Respiratory alkalosis
c) Gastric outlet obstruction
d) Hirschsprung Diseasee) Gastric atresia 25. At moderate pneumonia antibacterial
therapy lasts:
19. A neonate presents with cyanosis which a) 5 - 7 days
fails to improve with inhaled oxygen. ECG b) 7 - 10 days
shows left axis deviation. c) 10 - 14 day
The diagnosis is: d) 14 - 20 days
a) Hypoplastic Left Heart Syndrome e) 3 - 5 days
b) Mitral stenosis
c) Tetrology of Fallots 26. The etiological agents of hospital
d) Tricuspid Atresia pneumonia are:
e) Aortic stenosis a) E. coli, proteus
b) Brucella, streptococcus
20. The causes of death in protein energy c) Pneumococcus, tuberculosis
malnutrition: d) Staphylacoccus, influenza
a) Anemic heart failure e) Staphylacoccus, E. coli
b) Constipation
c) Bleeding 27. The beginning of obstructive bronchitis
d) Congenital heart failure is:
e) Loose stool a) Catarrhal syndrome
b) Allergic reactions
21.In the biochemical analysis of blood c) Inspiration dyspnea
сhild with malnutrition will be: d) Expiration dyspnea
a) Hyperglycemia e) Cough, temperature
b) Hypercalcemia
c) Hyperalbuminemia 28. Which main clinical features are useful
d) Hypoproteinemia in the diagnosis of bronchiolitis:
e) Hypoproteinemia and hypercalcemia a) Paroxysmal cough
b) Wheezing
22. The chest X-ray sign typical for acute c) Tachypnea
pneumonia is: d) Dyspnea
a) Strengthening of pulmonary picture (lung e) Inspiration dyspnea
pattern)
29.Bronchitis is caused most often by: 35. What is the most common congenital
a) Fungi heart defect with a left to right shunt causing
b) Viruses congestive heart failure in the
c) Bacteria pediatric age group?
d) Parasites a) Atrial septal defect
e) Allergens b) Atrioventricular canal
c) Ventricular septal defect
30. What is the most common congenital d) Patent ductus arteriosus
heart defect with a left to right shunt causing e) Tetrology of Fallots
congestive heart failure in the
pediatric age group? 36. What is the main difference between
a) Atrial septal defect Rheumatic Heart Disease (RHD) and Acute
b) Atrioventricular canal Rheumatic Fever (ARF)?
c)Ventricular septal defect a) In ARF there is an elevated ESR
d)Patent ductus arteriosus b) In RHD there is a prolonged P-R interval
e) Tetrology of Fallots c) In ARF there is a history of arthralgias
d) In RHD there is evidence of chronic heart
31. The main symptom of (PDA) patent disease
ductus arteriosus in children is: e) In ARF there is a history of carditis
a) Tachycardia or other arrhythmia
b) Shortness of breath and other respiratory 37. The three D's of pellagra are:
problems a) diarrhea, dementia, dermatitis
c) Continuous machine-like murmur b) diuresis, dilation of bladder, dementia
d) Enlarged heart c) detoxicaton, dizziness,deafness
e) Cough d) demineralization, dementia, dizziness
e) dementia, dizziness, dermatitis
32. Which heart murmur is characteristic for
(PDA) patent ductus arteriosus in children? 38. Child 10 years old complaine on loss of
a) Systolic appetite, heartburn, pain in the navel, which
b) Diastolic is dull and occurs more often
c) Systolic and diastolic within 2-3 hours after eating and in the
d) morning - on an empty stomach. The pain
e) Continuous murmur decreases after a meal. Ну is sick
f) Systolic and diastolic for three years. OBJECTIVELY: Skin is
pale. Abdomen is soft, pain during deep
33. What is the criterion of carditis in acute palpation in the epigastric and
rheumatic fever ARF in children? pyloroduodenal areas. Positive Mendel
a) Damage of myocardium and symptom. What is the most probable
endocardium diagnosis?
b) Damage of cardialgias a) Chronic cholecystocholangitis
c) Damage of epicardium b) Crohn's Disease
d) Damage of pericardium c) Chronic gastroduodenitis
e) Damage of pericardium and endocardium d) Peptic ulcer
e) Chronic gastritis
34. Common causes of mortality in primary
nephrotic syndrome is/are: 39. What is the main criterion of acute
a) Acute renal failure rheumatic fever (ARF) in children?
b) Congestive heart failure a) Carditis
c) Peritonitis b) Hepatitis
d) Seizure c) Nephritis
e) Carditis d) Pneumonia
e) Dermatitis

40. Paratrofia in children is manifested by:


a) Decreased body weight proportionally to ultrasound: thickened wall and sealed
the length intrahepatic bile ducts and gall bladder.
b) Increased body weight proportionally to What is the main diagnosis you set sick?
the length a) Acute cholecystitis.
c) Increased body weight in stunted growth b) Chronic cholecystitis.
d) Decreased body weight with an c) Chronic cholecystocholangitis.
accelerated growth d) Biliary dyskinesia hypokinetic by type
e) Decreased body weight with an e) Biliary dyskinesia hypertensive type
accelerated length
45. In the 12-year-old boy there is aching
41. For giardiasis, all the symptoms are pain in the epigastrium after 1,5 hours after
characteristic, except: eating and fasting, periodic
a) Loss of appetite nausea, vomiting, heartburn. Recently, the
b) Itching child gets tired quickly, complaining of
c) Rumbling and bloating headache, weakness. Chronic antral
d) Children have a delay in physical gastritis was diagnosed due to results of
development clinical and additional methods of
e) Nausea examinations. What clinical syndromes
are the leading?
42. With peptic ulcer of the stomach, "early a) Pain, dyspepsial, asthenic-vegetative
pain" appears after taking food through: b) Asthenic, pain, cephalgic
a) 15-30 min c) Pain, intoxication, asthenic
b) 60-70 minutes d) Dyspepsial, ossalgic, intoxication
c) 60-120 min e) Pain, intoxication, intoxication
d) 5 min
e) 1 min 46. The patient, 15 years old, 5 years
suffering from duodenal ulcer. After
43. The patient, 15 years old, 5 years exercise felt severe pain in the epigastric
suffering from duodenal ulcer. After region and right upper quadrant. It was
exercise felt severe pain in the epigastric vomiting. There is a weak pulse, sharp pain
region and right upper quadrant. It was from the navel to the right, the
vomiting. There is a weak pulse, sharp pain disappearance of hepatic dullness during
from the navel to the right, the percussion. What caused condition that
disappearance of hepatic dullness during requires emergency care?
percussion. What caused condition that a) Duodenal ulcer perforation.
requires emergency care? b) Spontaneous pneumothorax
a) Spontaneous pneumothorax. c) Aggravation of kidney stones (renal colic)
b) Duodenal ulcer perforation. d) Exacerbations of cholelithiasis
c) Aggravation of kidney stones (renal e) Ulcer bleeding
colic).
d) Exacerbations of cholelithiasis 47. A 10 years old boy has acute
e) Ulcer bleeding glomerulonephritis during a mouth. He has
edema. In urine: protein – 2,5 g/l, in the
44. A girl, 12 years, complains of biochemical blood test: total protein – 48 g/l,
paroxysmal pain in the right upper quadrant, cholesterol- 9,8 mmol|l. What from this
which increases during exercise, after medicine must be appointed to
eating fatty foods, nausea, loss of appetite, the child in the complex of pathogenetical
headache, fatigue. Sick for 5 years. These therapy?
objective examination: skin pale a) Delagyl
tongue with yellow-gray stratification. b) Plaquenyl
Abdomen soft, painful in the right upper c) Prednisolon
quadrant, positive symptoms Ortner, d) Heparin
Murphy, Kera, the liver acts from the edge e) Curantil
of costal arch to 2 cm edge sharp. The
tendency to constipation. These
48. The boy of 3 has an edema syndrome subfebrility. A positive symptom of Kera. In
like as anasarca. Blood pressure - 95/60. In blood tests, an increase in ESR is noted.
the general analysis of urine: What disease causes such a
protein – 6,3 g\l, leucocytes 2-3 in field of clinical picture:
view, red corpuscles 1-2 in field of view, a) Chronic enterocolitis
cylinders - 2-3 in field of view. b) Chronic cholecystitis
General protein of blood – 44,2 g\l, albumen c) Chronic duodenitis
– 38,1%, cholesterol of blood – 8,6 mmol\l. d) Biliary dyskinesia
What clinical variant of e) Acute intestinal infection
acute glomerulonephritis does take place
probably? 52. A 1.5-year-old child was admitted to the
a) Acute glomerulonephritis with nephritic hospital on the 3rd day of the disease. He
syndrome became acutely ill, the
b) Acute glomerulonephritis with nefrotic temperature was 39.2 C, weakness,
syndrome coughing, refusal to eat, shortness of breath
c) Acute glomerulonephritis with isolated appeared from the 3rd day, cough
urine syndrome intensified. Upon admission: adynamic, pale,
d) Acute glomerulonephritis with nefrotic cyanosis of the nasolabial triangle,
syndrome, hematuria and arterial temperature - 38.5 C, respiratory
hypertension rate - 52 per minute. Above the lungs -
e) Acute glomerulonephritis with hematuria shortening of the sound in the right scapular
region. On the R-gram:
49. A 12-year-old boy is registered at a homogeneous infiltration of segments 8-10
dispensary with a cardiorheumatologist with on the right. Make a diagnosis.
a diagnosis of rheumatism, a) Obstructive bronchitis
active phase, rheumacarditis with mitral b) Bronchitis
valve damage. How long should secondary c) Interstitial pneumonia
bicillin-drug prevention of d) Bronchiolitis
rheumatism be carried out? e) Segmental pneumonia
a) for life
b) 3 years 53. Identify the following diagnoses is most
c) 1 year probable. A boy 12 years old admitted to the
d) 18 years old hospital with intermittent
e) 25 years high fever, allergic rash, pain and swelling
in the knee and ankle joints, increase of
50. Pneumonia during auscultation is peripheral lymph nodes, liver
characterized by: and spleen. In blood test - leukocytes
a) The presence of dry wheezing, which 27x109/l, ESR - 65mm/hour, increased
increases on exhalation, wheezing breathing immunoglobulin M and G.
b) The presence of crepitation or small- a) Sepsis
bubbly wet wheezing, weakened or b) Systemic lupus erythematosus
bronchial breathing c) Systemic juvenile rheumatoid arthritis
c) The presence of dry wheezing, scattered d) Rheumatc fever
throughout the pulmonary fields, hard e) Leukemia
breathing
d) The presence of small bubbly wheezes at 54. Put a preliminary diagnosis. Patient
the height of inspiration, various wet and 14yrs old complained of intense pain in the
dry wheezes, which change the right lumbar region, chills,
character after coughing accompanied by fever up to 39°C. The
e) The presence of wet and dry wheezing abdomen is soft, painful in the right area.
Palpation of right kidney is
51. A 8-year-old child has been ill for three painful. In the blood: leukocytes 30.0
years. Complaints of abdominal pain, more x109/L, ESR - 50 mm/hour. In urine an.:
in the right hypochondrium, acid reaction, leukocytes in the
entire field of vision. According to the US - periorbital region initially and eventually
the left kidney is normal, the contours of the spreads
right kidney are to the rest of the body. The patient is given
increased. steroid therapy
a) Right paranephritis and the disease goes away.What is a key
b) Acute right-sided pyelonephritis morphological feature of the patient’s
c) Tuberculosis of the right kidney disease?
d) Swelling of the right kidney a) Fusion of the foot processes
e) Polycystic kidney degeneration b) Destruction of the basement membrane
c) Destruction of the glomerulus
55. Find the disease that led to the current d) Hemosiderin laden macrophages in the
state of the patient.13 years old girl was kidney
hospitalized with straining pain e) None of the above
in the left hypochondrium, which irradiates
to the back. He notes nausea, decreased 59. Against the background of acute
appetite, weight loss, vomiting respiratory viral infections, an 11-year-old
without relief, diarrhea. He has been ill for child developed a dry cough. During
over 5 years. Exacerbation has developed auscultation, hard breathing, dry and single
because of errors in the diet. medium-bubbly diffuse wheezing on
Objective: t ° = 37,0°C, pulse rate 94 per inspiration, decreasing after
minute, BP 125/75. Skin is pale, pain in the coughing, is determined. Radiologically
epigastrium, right and left symmetrical enhancement of the pulmonary
hypochondrium. In the blood test: Leuk. pattern in the basal zones. In the
10.4 x 109/l, ESR 22 mm/hour. general blood test – relative lymphocytosis.
a) Stomach ulcer Specify the probable diagnosis.
b) Chronic gastritis a) Acute obstructive bronchitis
c) Chronic cholecystitis b) Acute simple bronchitis
d) Chronic pancreatitis c) Bronchiolitis
e) Chronic enterocolitis d) Pneumonia
e) Recurrent bronchitis
56. Indicate the drug which should be
prescribed primarily for treatment. 6 years 60. A 1.5-year-old child was admitted to the
old boy complains of an acute hospital on the 3rd day of the disease. He
abdominal pain, which arises after mental became acutely ill, the
loading, use of cold drinks, ice-cream. The temperature was 39.2 C, weakness,
diagnosis: Dyskinesia of coughing, refusal to eat, shortness of breath
gallbladder, hypertonic type. appeared from the 3rd day, cough
a) Spasmolitics and choleretics intensified. Upon admission: adynamic, pale,
b) Sedative and cholikinetics cyanosis of the nasolabial triangle,
c) Choleretics and cholikinetics temperature - 38.5 C, respiratory rate
d) Antioxidants - 52 per minute. Above the lungs -
e) Antibiotics shortening of the sound in the right scapular
region. On the R-gram: homogeneous
57. Which of the following is the treatment infiltration of segments 8-10 on the right.
of choice for peptic ulcer disease caused by Make a diagnosis.
H. pylori infection? a) Obstructive bronchitis
a) PPI, amoxicillin, and clarithromycin b) Bronchitis
b) H2 blockers c) Interstitial pneumonia
c) Erythromycin d) Bronchiolitis
d) PPI and erythromycin e)
e) PPI and azithromycin58. A 7-year-old Segmental pneumonia
child present with hypoalbuminemia,
edema,hyperlipidemia and proteinuria.The 61. Which of the following is the best next
edema is in the step after 8 weeks of proton pump inhibitor
trial without improvement
of chronic epigastric discomfort? H2 receptor antagonists, antibiotics,
a) Colonoscopy probiotics, antacids
b) H2 inhibitors 4.
c) Continue proton pump inhibitors H2 receptor antagonists, nonsteroidal anti-
d) Endoscopy with biopsy inflammatory drugs, proton pump inhibitors,
e) Abdominal ultrasonography mucosal protectants
5. Nonsteroidal anti-inflammatory drugs,
62. What is an effective test for detection of antibiotics, proton pump inhibitors, mucosal
H. pylori infection? protectants, probiotics
a) Ultrasound scan
b) Urea breath 66. Which of the following best describes
c) Blood picture the flow of bile from the liver to the
d) Urine analysis gallbladder and then the small
e) GFR (glomerular filtration rate) intestines?
1.
63. Which of the following medical Right and left hepatic duct - Common
conditions is the most common cause of hepatic duct - Cystic duct - Gallbladder -
dyspepsia in children? Cystic duct - Common bile
a. duct - Sphincter of Oddi
Functional dyspepsia 2.
b. Right and left hepatic duct - Common bile
Gastric cancer duct - Cystic duct - Gallbladder - Cystic
c. duct - Common hepatic
Peptic ulcer disease duct - Sphincter of Oddi
d. 3.
Pancreatitis Sphincter of Oddi - Right and left hepatic
e. duct - Common hepatic duct - Cystic duct -
Esophageal cancer Gallbladder - Cystic duct
- Common bile duct
64. Which of the following is the best next 4.
step in the management of Functional Right and left hepatic duct - Common bile
dyspepsia with alarm features in duct - Gallbladder - Cystic duct - Common
children? hepatic duct - Sphincter
1. of Oddi5.
Proton pump inhibitors Right and left hepatic duct - Gallbladder -
2. Common bile duct - Cystic duct - Liver -
Testing for Helicobacter pylori Cystic duct - Common
3. hepatic duct - Sphincter of Oddi
Endoscopy
4. 67. A positive Murphy's sign is highly
Diet and lifestyle modification indicative of acute cholecystitis. Which of
5. the following statements
H2 antagonists demonstrates this maneuver?
6.
65. What types of medication are considered Pain during inhalation as the examiner
for a client with peptic ulcer disease (PUD) places his/her hand over the rt. upper
in children? quadrant
1. 7.
H2 receptor antagonists, antibiotics, proton Pain during exhalation as the examiner
pump inhibitors, probiotics places his/her hand over the rt. upper
2. quadrant
H2 receptor antagonists, antibiotics, proton 8.
pump inhibitors, mucosal protectants Pain in rt. lower quadrant on palpation of
3. the lt. lower quadrant
9.
Pain during hyper-extension of the rt. hip 1.
joint Pure cholesterol stone
10. Pain on palpation over McBurney's 2.
point Mixed stones
3.
68. A patient has pain on the right side of Combined stones
the abdomen radiating to the right shoulder. 4.
The serum transaminases, Pigment stone
alkaline phosphatase, and serum lipase are 5. Uric acid stone
increased. Which location is the MOST
likely site of impaction of the 72. What respiratory rate might indicate that
stone? an infant has severe bronchiolitis in children?
1. 1.
The stone is present at the sphincter of Oddi. < 50 breaths/min
2. 2.
The stone is impacted at the cystic duct. 70 breaths/min
3. 3.
The stone is impacted at the left hepatic bile 40 breaths/min
duct. 4.
4. 30–40 breaths/min
The stone is impacted at the right hepatic 5.
duct. 30 breaths/min
5.
The stone is impacted at the neck of the 73. What kind of supports can help infants
gallbladder. manage viral bronchiolitis in children?
1. Antipyretics, nasal suction, hydration,
69. Which of the following is a supplemental oxygen
characteristic gallblabber abnormality? 2. Antibiotics, nasal suction, hydration,
1. stopping breastfeeding
Phrygian cap 3. Antipyretics, nasal suction, fluid
2. Absence of entire extrahepatic duct restriction, supplemental oxygen
system 4. Antibiotics, nasal suction, hydration,
3. Atresia of bile duct supplemental oxygen
4. Accessory bile duct 5. Antipyretics, stopping breastfeeding,
5. supplemental oxygen
Choledochal cyst
74. What is the age of a typical patient
70. Which of the following is the treatment presenting with bronchiolitis in children?
of choice for symptomatic cholelithiasis? 1. Under 5 years of age
1. 2.
Cholecystectomy Over 3 years of age
2. 3. Under 2 years of age
Observation 4.
3. Between 1-2 years of age
Sphincterotomy 5.
4. Ursodeoxycholic acid Between 2-3 years of age
5.
Cholecystokinetic drug 75. What are risk factors that contribute to
bronchiolitis in children? Select all that
71. A 16-year-old girl presents with right apply.
upper quadrant pain. She has a BMI of 32 1. Being premature, attending daycare,
kg/m2. She complains of being breastfed
feeling nauseated at times. What is the 2. Being premature, attending daycare,
probable type of gallbladder stone causing having heart or lung defects
the right upper quadrant pain?
3. Being premature, being the first child, 3.
having heart or lung defects History of acute onset of persistent cough
4. Being breastfed, attending daycare, for 1–3 weeks, no clinical signs of
having heart or lung defects pneumonia (fever, rales, tachypnea),
5. Older age, attending daycare, having a pleural effusion or empyema on chest
heart or lung defects radiograph
4.
76. Which of the following options provides History of acute onset of persistent cough
the correct order of lung structures from for 1–3 weeks, no clinical signs of
proximal to distal? pneumonia (fever, rales, tachypnea),
1. an elevated WBC count, in the range of
Terminal bronchioles, respiratory 15,000-40,000/mm3
bronchioles, alveolar sacs, alveolar ducts 5.
2. History of acute onset of persistent cough
Respiratory bronchioles, terminal for 1–3 weeks, clinical signs of pneumonia
bronchioles, alveolar ducts, alveolar sacs (fever, rales, tachypnea),
3. positive blood culture
Respiratory bronchioles, terminal
bronchioles, alveolar sacs, alveolar ducts4. 79. A patient diagnosed with pneumonia
Terminal bronchioles, respiratory was hospitalized for 10 days. In spite of an
bronchioles, alveolar ducts, alveolar sacs initial improvement with
5. administration of amoxicillin/clavulanic
Terminal bronchioles, alveolar ducts, acid and clarithromycin, by day 6 the
respiratory bronchioles, alveolar sacs patient's condition began to deteriorate.
Ticarcillin was added with good response,
77. 16-year-old male presents with a history and eventual resolution of symptoms. What
of productive cough and low grade fever. was the probable causative
He was previously healthy organism in this patient?
and does not suffer from any chronic 1.
ailments. He is diagnosed with Streptococcus pneumoniae
tracheobronchitis. What is the next step in 2.
management? Pseudomonas aeruginosa
1. 3.
Supportive care only Legionella pneumophila
2. 4.
Inhaled steroids Mycoplasma pneumoniae
3. 5.
Inhaled bronchodilators Haemophilus influenza
4.
Oral third generation cephalosporins 80. Which of the following best defines
5. hospital-acquired pneumonia in children?
Oral neuraminidase inhibitors 1.
Pneumonia acquired in the community
78. Which factors would indicate a requiring hospitalization
diagnosis of bronchitis in children? 2.
1. Pneumonia developed by healthcare
History of acute onset of persistent cough personnel
for 1–3 weeks, no clinical signs of 3.
pneumonia (fever, rales, tachypnea), Pneumonia associated with foreign body
an infiltrate on chest radiograph aspiration
2. 4.
History of acute onset of persistent cough Pneumonia acquired 48 hours after a
for 1–3 weeks, no clinical signs of hospital admission unrelated to pneumonia
pneumonia (fever, rales, tachypnea), 5.
chest X-ray to rule out pneumonia Pneumonia associated with medication
85. What age range of children should be
81. Which of the following symptoms treated with amoxicillin for bacterial
represent atypical pneumonia caused by pneumonia?
Mycoplasma pneumonia in 1.
children? Children 5–8 years old
1. Nonproductive cough, rhonchi, and 2.
myalgia Children < 5 years old
2. Nonproductive cough, wheezing, and 3.
myalgia Children 5–10 years old
3. 4.
Productive cough, rhonchi, and arthralgia Children > 5 years old5. Adolescents
4. Nonproductive cough, rhonchi, and
arthralgia 86. Which of the following is the antibiotic
5. of choice for bacterial pneumonia in an
Productive cough, wheezing, and arthralgia otherwise healthy child 1-4
years of age?
82. Which of the following is the most 1. Amoxicillin
common cause of atypical pneumonia in 2. Azithromycin
children? 3.
1. Clindamycin
L. pneumophila 4.
2. Moxifloxacin
M. pneumoniae 5. Vancomycin
3.
C. pneumoniae 87. Which of the following is the bacterial
4. pathogen most commonly found in cases of
Respiratory syncytial virus community acquired
5. pneumonia?
S. aureus 1.
Streptococcus pneumoniae
83. What is the indication for doing a blood 2.
culture in patients with pneumonia in Group A Streptococci
children? 3.
1. Group B Streptococci
Routine work-up for pneumonia 4.
2. Staphylococcus aureus
Presence of hemoptysis 5.
3. Negative chest x-ray findings Staphylococcus epidermidis
4. Non-productive cough
5. 88. Which sign or symptom would suggest a
Suspicion of sepsis primary viral pneumonia due to influenza
over a typical influenza
84. Which of the following is a common illness?
cause of acute pneumonia, especially in the 1.
pediatric population? Dyspnea
1. 2.
Mycoplasma pneumonia Consolidative sounds on auscultation
2. Nocardia 3.
3. Actinomycosis Fever
4. 4.
Tuberculosis Productive cough
5. Aspergillus 5.
Significant myalgias
89. Which of the following drugs is added Inspection, palpation, percussion, blood
for the treatment of infants with pneumonia pressure
due to Chlamydia 2.
trachomatis? Inspection, palpation, percussion,
1. auscultation
Ciprofloxacin 3.
2. Azithromycin Inspection, pulse volume, percussion,
3. auscultation
Clindamycin 4.
4. Amoxicillin Inspection, palpation the liver, percussion,
5. auscultation
Moxifloxacin 5.
Inspection, palpation, retraction,
90. What types of pathogens typically cause auscultation
pneumonia in children 0 to 18 years old?
1. 93. Which neonatal infection is caused by
Mycoplasma pneumoniae, Haemophilus Chlamydia trachomatis 2–3 weeks after
influenzae, Chlamydia pneumoniae, birth?
Adenoviruses 1. Asthma
2. 2.
Streptococcus pneumoniae, Haemophilus Emphysema
influenzae, Chlamydia pneumoniae, 3.
Influenza virus A and B Bronchitis
3. 4.
Mycoplasma pneumoniae, Haemophilus Pneumonia
influenzae, Chlamydia pneumoniae, 5.
Escherichia coli Bronchiectasis
4.
Streptococcus pneumoniae, Mycoplasma 94. Which of the following cardiac diseases
pneumoniae, Haemophilus influenzae, is commonly caused by acute rheumatic
Chlamydia pneumoniae fever?
5. 1.
Staphylococcus aureus, Haemophilus Myocardial infarction
influenzae, Chlamydia pneumoniae, 2.
Influenza virus A and B Infectious endocarditis
3.
91. Which of the following test has a rapid Mitral stenosis
response time and is reliable in providing 4.
the diagnosis of mycoplasma Mitral valve prolapse
pneumonia? 5.
1. Pulmonic stenosis
Polymerase chain reaction (PCR)
2. 95. In acute rheumatic fever, antibodies
Mycoplasma antibody rise in titres against group A streptococcus cross-react
3. with antigens present on which
Sputum culture of the following structures of the heart?
4. 1.
Chest x-ray Epicardium and myocardium
5. 2.
Gram stain on sputum sample Myocardium and valves
3.
92. What assessments are part of a full Epicardium and endocardium
respiratory examination? 4.
1. Epicardium and valves
5.
Endothelium and epithelium96. Which of Sore throat
the following is most likely the main 5.
indication for antibiotic treatment in Prolonged P-R interval
children with group A
Streptococcus pharyngitis? 100. Which of the following is a major
1. criterion used to diagnose acute rheumatic
Reducing symptom severity fever?
2. 1.
Prevention of post-streptococcal Erythema multiforme
glomerulonephritis 2.
3. Fever
Reducing symptom duration 3.
4. Sore throat
Prevention of acute rheumatic fever 4.
5. Prolonged P-R interval
Prevention of spread to others 5.
Subcutaneous nodules
97. Which of the following is a major
criterion used to diagnose acute rheumatic 101. Which of the following is a major
fever? criterion used to diagnose acute rheumatic
1. fever?
Carditis 1.
2. Erythema multiforme
Erythema multiforme 2.
3. Fever
Sjogren’s syndrome 3.
4. Sore throat
Sore throat 4.
5. Chorea
Hepatitis 5.
Prolonged P-R interval
98. Which of the following is a major
criterion used to diagnose acute rheumatic 102. Which of the following is a minor
fever? criterion used to diagnose acute rheumatic
1. fever?
Erythema multiforme 1.
2. Fever
Polyarthritis 2.
3. Chorea
Sjogren’s syndrome 3.
4. Erythema multiforme
Sore throat 4.
5. Sore throat
Prolonged P-R interval 5.
Subcutaneous nodules
99. Which of the following is a major
criterion used to diagnose acute rheumatic 103. Which of the following is a minor
fever? criterion used to diagnose acute rheumatic
1. fever?
Erythema marginatum 1.
2. Prolonged P-R interval
Erythema multiforme 2.
3. Chorea
Fever 3.
4. Erythema multiforme
4. 107. What is the underlying
Sore throat pathophysiology of mitral valve damage in
5. acute rheumatic fever?
Subcutaneous nodules 1.
Cross-reactive antibodies targeting valve
104. Which of the following is a minor tissue
criterion used to diagnose acute rheumatic 2.
fever? Congenital mucinous infiltration of valve
1. leaflets3.
Chorea Bacterial colonization of abnormal valve
2. tissue
Erythema multiforme 4.
3. Infectious endocarditis
Sore throat 5.
4. Arthralgia Reactivation of latent infectious organisms
5.
Subcutaneous nodules 108. Which of the following valvular
abnormalities is least likely to be a direct
105. Which of the following is a minor result of acute rheumatic heart
criterion used to diagnose acute rheumatic disease?
fever? 1.
1. Pulmonary stenosis
Elevated acute-phase reactants (erythrocyte 2.
sedimentation rate; C-reactive protein) Mitral stenosis
2. 3.
Supporting evidence of antecedent group A Mitral regurgitation
streptococcal infection 4. Aortic stenosis
3. 5. Aortic regurgitation
Carditis
4. 109. Which of the following describes the
Polyarthritis murmur of mitral stenosis?
5. 1.
Subcutaneous nodules Opening snap followed by a rumbling mitral
diastolic murmur.
106. A 12-year-old boy has sore throat, 2.
fever, tender and swollen anterior cervical Mid-systolic click with a mid-diastolic
nodes, and bilateral tonsillar rumble
exudates for 2 days. He has no coryza, 3.
conjunctivitis, cough, hoarseness, anterior Systolic crescendo-decrescendo murmur
stomatitis, discrete ulcerative 4.
lesions or vesicles, or diarrhea. What is the Early diastolic decrescendo murmur
next best step in management? 5.
1. High-pitched holosystolic murmur
Initiate antibiotics
2. 110. Which of the following is true
Obtain blood cultures regarding mitral stenosis?
3. 1.
Obtain a rapid streptococcal antigen test Early-diastolic murmur, difficulty in closing
4. 2.
Prescribe acetaminophen Diastolic murmur, difficulty in opening
5. 3.
Refer for hospitalization Holosystolic murmur, difficulty in opening
4.
Late systolic murmur, difficulty in opening
5.
Crescendo-decrescendo, floppy valve 4. Atriums
5.
111. Which of the following is the most Right atrium
common cause of mitral stenosis?
1. 115. Which one of the listed conditions
Dystrophic calcification leads to left to right shunting?
2. 1.
Bicuspid valve Pulmonary stenosis
3. 2. Atherosclerotic heart disease
Congenital mitral stenosis 3.
4. Patent ductus arteriosus
Rheumatic heart disease 4.
5. Coarctation of the aorta
Intravenous drug use 5. Aortic stenosis

112. Which part of the heart is often 116. Which one of the listed conditions
enlarged due to mitral stenosis? leads to left to right shunting?
1. 1. Atrial septal defect
Left atrium 2.
2. Pulmonary stenosis
Left ventricle 3. Atherosclerotic heart disease
3. 4.
Right atrium Coarctation of the aorta
4. Aorta 5. Aortic stenosis
5.
Right ventricle 117. Which of the following is the most
common type of atrial septal defect?
113. How would you differentiate mitral 1.
regurgitation from mitral stenosis in a gross Sinus venosus
pathologic specimen of the 2.
heart? Ostium secundum
1. 3.
The left atrium and left ventricle are Ostium primum
enlarged in mitral regurgitation. 4.
2. Coronary sinus
The left atrium is atrophied in mitral 5. Atrioventricular canal defect
stenosis.
3. 118. Classic continuous machinery murmur
The left ventricle is hypertrophic in mitral is typically heard in which of the following
stenosis. conditions?1. Aortic stenosis
4. 2. Ventricular septal defect
The left atrium is atrophic in mitral 3.
regurgitation. Patent ductus arteriosus
5. 4.
The interventricular septum is enlarged in Coarctation of aorta
mitral stenosis 5. Atrial septal defect

114. Aortic stenosis directly affects which 119. Ventricular septal defect has which of
of the given chambers of the heart? the following types of murmurs?
1. 1.
Left ventricle Pansystolic murmur
2. 2.
Right ventricle Still’s murmur
3. 3.
Left atrium Mid diastolic murmur
4. 124. What condition may result from a long-
Early diastolic murmur standing, left-to-right shunting of blood,
5. which involves reversal into a
Ejection systolic murmur cyanotic right-to-left shunt?
1.
120. Which one of the following is Eisenmenger's syndrome
considered a congenital cyanotic heart 2. Aortic stenosis
disease? 3.
1. Pulmonic stenosis
Tetralogy of Fallot 4.
2. Mitral stenosis
Coarctation of the aorta 5.
3. Tricuspid stenosis
Cardiomyopathy
4. 125. Which of the given heart diseases
Pericardial effusion mainly causes obstruction to blood flow
5. from the left ventricle?
Cardiomegaly 1.
Truncus arteriosus
121. Which of the following allows blood to 2.
bypass the pulmonary circulation during Patent ductus arteriosus
fetal life? 3. Atrial septal defect
1. Ventricular septum 4. Aortic stenosis
2. 5.
Ductus arteriosus Transposition of great arteries
3. Atrial septum
4. Atrioventricular canal 126. Which of the following conditions is
5. most likely to present with cyanosis?
Truncus arteriosus 1.
Transposition of the great arteries
122. Which of the following congenital 2. Atrial septal defect
heart lesions is most likely to require 3. Ventricular septal defect
surgical attention in early childhood? 4.
1. Atrial septal defect Coarctation of the aorta
2. 5.
Patent ductus arteriosus Bicuspid aortic valve
3.
Pulmonic stenosis 127. What is the most common congenital
4. heart defect?
Transposition of the great arteries leading to 1. Atrial septal defect
blue baby 2.
5. Ventricular septal defect Patent foramen ovale
3.
123. Which of the following congenital Mitral atresia
heart lesions results in persistent blood flow 4. Ventricular septal defect
directly from the aorta into the 5.
pulmonary artery? Tricuspid atresia
1.
Patent ductus arteriosus 128. What heart defect prevents blood flow
2. Atrial septal defect from the right atrium to the right ventricle?
3. Ventricular septal defect 1.
4. Tricuspid atresia
Coarctation of the aorta 2.
5. Mitral atresia
Pulmonic stenosis 3. Ventricular septal defect
4. 1.
Transposition of the great arteries Epinephrine drip
5. Atrial septal defect 2.
Bicarb infusion
129. What is the classical radiologic finding 3.
in tetralogy of Fallot? Surfactant
1. 4.
Boot-shaped heart Indomethacin
2. 5.
Egg-on-a-string sign Prostaglandins
3.
Snowman sign 134. Which of the following is a common
4. finding in aortic coarctation?
Scimitar sign 1.
5. Hypertension
Gooseneck sign 2.
Left atrial hypertrophy
130. In coarctation of the aorta, what would 3.
be the expected quality of the pulses and Patent ductus arteriosus
blood pressure? 4.
1. Notably decreased in vessels before point Right ventricular hypertrophy
of coarctation 5.
2. Notably increased in vessels below the Tachycardia
coarctation3. Notably decreased in vessels
below the coarctation 135. Which of the following is characteristic
4. Unaffected by the coarctation of aortic coarctation?
5. Notably increased only in collateral 1.
circulation Diminished radial pulses
2.
131. Which of the following medications Increased ankle brachial index
should be administered to keep the patent 3. Narrow pulse pressure
ductus arteriosus open? 4.
1. Diminished femoral pulses
Indomethacin 5.
2. Bifid carotid pulse
Prednisolone
3. 136. Which of the following is a more
Montelukast accurate description of murmur associated
4. Atropine with aortic stenosis?
5. 1. A systolic ejection murmur heard best at
Prostaglandin E1 the apex
2. A crescendo-decrescendo diastolic
132. The oxygenated blood from the murmur heard best at the apex
placenta is delivered to the fetus via what 3. A systolic ejection murmur heard best at
vessel? the base of the heart
1. Umbilical vein 4. A crescendo-decrescendo diastolic
2. Vitelline vein murmur heard best at the base of the heart
3. Umbilical artery 5. A constant murmur heard in both systole
4. Anterior cardinal vein and diastole
5.
Posterior cardinal vein 137. Which of the following features will
allow a baby with transposition of the great
133. What is the first medication you give a vessels to remain alive at
baby with suspected ductal dependent birth?
cyanotic heart disease? 1.
Patent ductus arteriosus 2.
2. Tetralogy of Fallot
Mitral valve stenosis 3.
3. Transposition of the great arteries
Pulmonary atresia 4. Ventricular septal defect
4. An arteriovenous malformation in the 5. Aortic stenosis
lungs
allowing for shunting 141. Which of the following cardiac
5. diseases is acyanotic congenital heart lesion
Surfactant administration at birth causing a pressure load?1.
Coarctation of the aorta
138. A 17-year-old student and football 2.
player is referred to your primary care office Tetralogy of Fallot
for evaluation of high blood 3.
pressure. During an annual check-up, he Transposition of the great arteries
was noted to have an arterial blood pressure 4. Aortic stenosis
of 155/90 mm Hg without 5.
presenting symptoms. On physical Patent ductus arteriosus
examination, his blood pressure is 152/94
mm Hg, and his heart rate is 59 142. Which of the following is a diagnostic
beats/min. On auscultation, a systolic criterion for acute kidney injury (AKI)?
ejection murmur is heard over the spine 1.
between the shoulder blades. Of note Persistent evidence of kidney damage or a
is that his femoral pulses are not well felt. decrease in function for at least 3 months
His laboratory test results are normal; 2.
however, an ECG indicates left Hematuria with dysmorphic red blood cells
ventricular hypertrophy. Which of the 3.
following is the most likely diagnosis? Oliguria (urine volume ≤ 0.5 mL/kg/hr for 6
1. hr)
Coarctation of the aorta 4.
2. Aortic stenosis Heavy proteinuria (protein excretion greater
3. than 3.5 g/24 hours)
Bicuspid aortic valve 5.
4. Multiple cysts on renal imaging
Hypertrophic obstructive cardiomyopathy
5. 143. Which of the following conditions is
Patent ductus arteriosus characterised by an abrupt loss of kidney
function leading to a rapid
139. Which of the following cardiac decline in the glomerular filtration rate
diseases is acyanotic congenital heart lesion (GFR), accumulation of waste products such
causing a volume load? as blood urea nitrogen (BUN)
1. and creatinine, and dysregulation of
Coarctation of the aorta extracellular volume and electrolyte
2. homeostasis?
Tetralogy of Fallot 1. Acute kidney injury
3. 2.
Transposition of the great arteries Chronic kidney disease
4. Atrial septal defect 3.
5. Aortic stenosis Pyelonephritis
4. Vesicoureteral reflux
140. Which of the following cardiac 5.
diseases is acyanotic congenital heart lesion Renal stones
causing a volume load?
1.
Coarctation of the aorta
144. Which of the following patients would 147. Which of the following is a diagnostic
be diagnosed with stage G1 chronic kidney criterion for acute kidney injury (AKI)?
disease? 1.
1. Persistent evidence of kidney damage or a
Patient B: irreversible kidney damage; GFR decrease in function for at least 3 months
65 mL/min per 1.73 m2) 2.
2. Hematuria with dysmorphic red blood cells
Patient C: irreversible kidney damage; GFR 3.
50 mL/min per 1.73 m2) Heavy proteinuria (protein excretion greater
3. than 3.5 g/24 hours)
Patient D: irreversible kidney damage; GFR 4.
44 mL/min per 1.73 m2) Multiple cysts on renal imaging
4. 5.
Patient A: irreversible kidney damage; GFR Increase in serum creatinine by ≥ 0.3 mg/dL
95 mL/min per 1.73 m2) from baseline within 48 hr
5.
Patient E: irreversible kidney damage; GFR 148. Which of the following is a diagnostic
14 mL/min per 1.73 m2) criterion for acute kidney injury (AKI)?
1.
145. Which of the following patients would Persistent evidence of kidney damage or a
be diagnosed with stage G2 chronic kidney decrease in function for at least 3 months
disease? 2.
1. Hematuria with dysmorphic red blood cells
Patient A: irreversible kidney damage; GFR 3.
65 mL/min per 1.73 m2) Heavy proteinuria (protein excretion greater
2. than 3.5 g/24 hours)
Patient B: irreversible kidney damage; GFR 4.
95 mL/min per 1.73 m2) Increase in serum creatinine to ≥ 1.5 times
3. baseline within the prior 7 days
Patient C: irreversible kidney damage; GFR 5.
50 mL/min per 1.73 m2) Multiple cysts on renal imaging
4.
Patient D: irreversible kidney damage; GFR 149. Which of the following conditions is
44 mL/min per 1.73 m2) characterised by lower urinary tract
5. symptoms (dysuria, urgency, new
Patient E: irreversible kidney damage; GFR onset urge incontinence, frequency, lower
14 mL/min per 1.73 m2) abdominal pain) with no fever or low-grade
146. Which of the following patients would fever (<38 0C) together with
be diagnosed with stage G3a chronic kidney a significant growth of bacteria on urine
disease? culture?
1. 1. Acute cystitis
Patient B: irreversible kidney damage; GFR 2.
95 mL/min per 1.73 m2) Chronic kidney disease
2. 3. Acute pyelonephritis
Patient A: irreversible kidney damage; GFR 4. Vesicoureteral reflux
50 mL/min per 1.73 m2) 5.
3. Renal stones
Patient C: irreversible kidney damage; GFR
65 mL/min per 1.73 m2) 150. Which of the following conditions is
4. characterised by fever with or without
Patient D: irreversible kidney damage; GFR abdominal pain, loin pain together
44 mL/min per 1.73 m2) with a significant growth of bacteria
5. (usually a single organism) on urine culture?
Patient E: irreversible kidney damage; GFR 1. Acute pyelonephritis
14 mL/min per 1.73 m2) 2. Acute cystitis3.
Chronic kidney disease 154. Which of the following are the
4. Vesicoureteral reflux common causes of acute tubular necrosis
5. (ATN) in critically ill infants and
Renal stones children?
1.
151. Which of the following are the Systemic lupus erythematosus
common causes of prerenal acute kidney 2.
injury? Severe and prolonged ischemic/hypoxic
1. injury and nephrotoxic insult
Glomerulonephritis: postinfectious; lupus 3.
erythematosus; membranoproliferative Posterior urethral valves, ureteropelvic
2. junction obstruction, urolithiasis
Hemolytic-uremic syndrome, acute tubular 4. Urethral strictures, Hemorrhagic cystitis,
necrosis, cortical necrosis Neurogenic bladder
3. 5. Acute poststreptococcal
Dehydration, sepsis, cardiac failure glomerulonephritis
4. Acute interstitial nephritis, tumor lysis
syndrome, toxin and drugs 155. What is vesicoureteral reflux grade 2?
5. 1.
Posterior urethral valves, ureteropelvic Reflux only fills the ureter without dilation.
junction obstruction, urolithiasis 2.
Reflux fills and mildly dilates the ureter and
152. Which of the following are the the collecting system.
common causes of postrenal acute kidney 3.
injury? Reflux fills and grossly dilates the ureter
1. and the collecting system with blunting of
Posterior urethral valves, ureteropelvic the calices.
junction obstruction, urolithiasis 4.
2. Reflux fills the ureter and the collecting
Dehydration, sepsis, cardiac failure system without dilation.
3. 5.
Glomerulonephritis: postinfectious; lupus Massive reflux grossly dilates the collecting
erythematosus; membranoproliferative system. All the calices are blunted with a
4. loss of papillary
Hemolytic-uremic syndrome, acute tubular impression, and intrarenal reflux may be
necrosis, cortical necrosis present. There is significant ureteral dilation
5. Acute interstitial nephritis, tumor lysis and tortuosity.
syndrome, toxin and drugs
156. What is vesicoureteral reflux grade 3?
153. Which of the following are the 1.
common causes of intrinsic renal acute Reflux fills and mildly dilates the ureter and
kidney injury? the collecting system.
1. 2.
Posterior urethral valves, ureteropelvic Reflux only fills the ureter without dilation.
junction obstruction, urolithiasis 3.
2. Reflux fills the ureter and the collecting
Dehydration, sepsis, cardiac failure system without dilation.
3. Urethral strictures, Hemorrhagic cystitis, 4.
Neurogenic bladder Reflux fills and grossly dilates the ureter
4. and the collecting system with blunting of
Glomerulonephritis: postinfectious; lupus the calices.
erythematosus; membranoproliferative 5.
5. Massive reflux grossly dilates the collecting
Gastroenteritis, Hemorrhage, Burns, system. All the calices are blunted with a
Anaphylaxis loss of papillary
impression, and intrarenal reflux may be Critically ill patients > 5 years, hypertension,
present. There is significant ureteral dilation fever
and tortuosity. 5. All children < 5 years, comorbid
conditions, hypertension
157. What is vesicoureteral reflux grade 4?
1. 160. What is pollakiuria (daytime frequency)
Reflux only fills the ureter without dilation. in a patient with voiding/ bladder
2. dysfunction?
Reflux fills and grossly dilates the ureter 1. Abnormally frequent small voids in a
and the collecting system with blunting of previously toilet-trained child with no
the calices. evidence of polyuria or urinary
3. tract infection
Reflux fills the ureter and the collecting 2. Uncontrolled leakage of urine, which can
system without dilation. be continuous or intermittent
4. 3.
Reflux fills and mildly dilates the ureter and The sudden and unexpected experience of
the collecting system. an immediate need to void
5. 4.
Massive reflux grossly dilates the collecting Difficulty in the initiation of voiding
system. All the calices are blunted with a 5.
loss of papillary Observed behavior used to either postpone
impression, and intrarenal reflux may be voiding or suppress urgency, including
present. There is significant ureteral dilation standing on tiptoe and
and tortuosity. forcefully crossing the legs

158. What is vesicoureteral reflux grade 5? 161. What is incontinence in a patient with
1. voiding/ bladder dysfunction?
Reflux only fills the ureter without dilation. 1. Abnormally frequent small voids in a
2. previously toilet-trained child with no
Reflux fills the ureter and the collecting evidence of polyuria or urinary
system without dilation. tract infection
3. 2.
Reflux fills and mildly dilates the ureter and The sudden and unexpected experience of
the collecting system. an immediate need to void
4. 3. Uncontrolled leakage of urine, which can
Massive reflux grossly dilates the collecting be continuous or intermittent
system. All the calices are blunted with a 4.
loss of papillary Difficulty in the initiation of voiding
impression, and intrarenal reflux may be 5.
present. There is significant ureteral dilation Observed behavior used to either postpone
and tortuosity. voiding or suppress urgency, including
5. standing on tiptoe and
Reflux fills and grossly dilates the ureter forcefully crossing the legs
and the collecting system with blunting of
the calices.159. What are the risk factors for 162. What is urgency in a patient with
pediatric acute kidney injury? voiding/ bladder dysfunction?
1. 1.
Critically ill patients, nephrotoxin use, The sudden and unexpected experience of
comorbid conditions an immediate need to void
2. All children < 5 years, fever, comorbid 2. Uncontrolled leakage of urine, which can
conditions be continuous or intermittent
3. 3. Abnormally frequent small voids in a
Critically ill adolescents, fever, comorbid previously toilet-trained child with no
conditions evidence of polyuria or urinary
4. tract infection
4. Sodium polystyrene sulfonate resin
Difficulty in the initiation of voiding (Kayexalate), 1 g/kg
5. 4.
Observed behavior used to either postpone Calcium gluconate 10% solution, 100
voiding or suppress urgency, including mg/kg/dose
standing on tiptoe and 5. Administration of hypertonic (3%) saline
forcefully crossing the legs
166. What is the treatment for severe
163. What is hesitancy in a patient with hyperkalemia (serum potassium >7 mEq/L)
voiding/ bladder dysfunction? in patients with acute kidney
1. Abnormally frequent small voids in a injury?
previously toilet-trained child with no 1. ACE inhibitors/angiotensin receptor
evidence of polyuria or urinary blockers (ARBs)2.
tract infection Calcium gluconate 10% solution, 100
2. mg/kg/dose
The sudden and unexpected experience of 3.
an immediate need to void Intravenous administration of isotonic saline,
3. Uncontrolled leakage of urine, which can 20 mL/kg over 30 min
be continuous or intermittent 4. Administration of hypertonic (3%) saline
4. 5.
Difficulty in the initiation of voiding Isradipine (0.05-0.15 mg/kg/dose)
5. 167. What is the treatment for anemia in
Observed behavior used to either postpone patients with chronic kidney disease?
voiding or suppress urgency, including 1.
standing on tiptoe and Erythropoiesis-stimulating agents
forcefully crossing the legs 2. ACE inhibitors/angiotensin receptor
blockers
164. What is holding maneuver in a patient 3.
with voiding/ bladder dysfunction? Recombinant human growth hormone
1. 4.
Difficulty in the initiation of voiding Phosphate binders
2. 5. Active vitamin D sterol agents (Calcitriol)
The sudden and unexpected experience of
an immediate need to void 168. What is the treatment for short stature
3. Uncontrolled leakage of urine, which can in children with CKD who remain less than
be continuous or intermittent −2 SD for height despite
4. optimal medical support (adequate caloric
Observed behavior used to either postpone intake and effective treatment of renal
voiding or suppress urgency, including osteodystrophy, anemia, and
standing on tiptoe and metabolic acidosis)?
forcefully crossing the legs 1.
5. Abnormally frequent small voids in a Erythropoiesis-stimulating agents
previously toilet-trained child with no 2. ACE inhibitors/angiotensin receptor
evidence of polyuria or urinary blockers
tract infection 3.
165. What is volume resuscitation in Recombinant human growth hormone
hypovolemic patients with prerenal acute 4.
kidney injury? Phosphate binders
1. 5. Active vitamin D sterol agents (Calcitriol)
Furosemide (2-4 mg/kg)
2. 169. Which of the following conditions is
Intravenous administration of isotonic saline, most likely to present with acute kidney
20 mL/kg over 30 min injury?
3. 1.
Pyelonephritis
2. Vesicoureteral reflux 2. Chronic cholecystitis
3. 3. Chronic duodenitis
Lupus nephritis 4. Biliary dyskinesia
4. 5. Acute intestinal infection
Diabetic nephropathy
5. 173. A 2-month-old baby with central
Cystitis cyanosis presented with the following
findings on chest X-ray and ECG.
170. A child of 2 months was delivered to What is the most probable underlying
the clinic on the 5th day from the onset of diagnosis?
the disease. He became acutely 1. Tetralogy of Fallot
ill: the temperature rose to 38.2 C, an 2. Hypertrophic left heart syndrome
unproductive cough appeared, the child 3. Coarctation of aorta
began to refuse food. Upon 4. Ebstein’s anomal
admission to the clinic, the temperature is 5. Hypertrophic right heart syndrome
38.5 C, BR = 70 per minute, shortness of
breath with a predominance of 174. A 7-year-old boy developed small
the expiratory component. The skin is pale, hypopigmented depressed scars after
perioral cyanosis is pronounced. There is a fingernail scratches; he has been treated
boxy sound above the lungs, with nonsteroidal antiinflammatory drugs
the breathing is hard, on both sides there is (NSAIDs) for a rheumatic disease 6 weeks
an abundance of small-bubbly wheezing on ago. Of the following, the
inhalation and exhalation. NSAID that is MOST likely to cause such a
No focal changes were detected on the X- unique skin reaction is
ray of the chest organs. Your diagnosis: A. Celecoxib
1. Aspiration pneumonia B. Meloxicam
2. C. Indomethacin
Bronchitis D. Naproxen
3. E. Ibuprofen
Bronchial asthma
4. 175. The gallbladder is congenitally absent
Foreign body in approximately 0.1% of the population.
5. Hypoplasia or absence of the
Bronchiolitis gallbladder can be associated with
1. Biliary dyskinesia
171. Cabbage, oatmeal and buckwheat 2. Cystic fibrosis
porridge, cheese, butter, baked potatoes, 3. Cirrhosis
raisins, prunes and pumpkin were 4. Cholestasis
included in the diet of a 9-year-old child. In 5. Wilson disease176. The recurrent
what disease is such a diet justified? aspiration of small quantities of gastric,
1. Peptic ulcer disease nasal, or oral contents can lead to several
2. Acute pneumonia clinical
3. Dysmetabolic nephropathy with presentations, including recurrent bronchitis
oxalaturia or bronchiolitis; recurrent pneumonia;
4. Rheumocarditis atelectasis; wheezing; cough;
5. Bronchial asthma apnea; and/or laryngospasm. The MOST
common underlying problem associated
172. A 9-year-old child has been ill for three with recurrent pneumonias in
years. Complaints of abdominal pain, more hospitalized children is
in the right hypochondrium, 1. Oropharyngeal incoordination
subfebrility. A positive symptom of Kera. In 2. Esophageal foreign body
blood tests, an increase in ESR is noted. 3. Nasoenteric tube
What disease causes such a 4. Poor oral hygiene
clinical picture: 5. Bronchopulmonary dysplasia
1. Chronic enterocolitis 177. Recurrent pneumonia is defined as:
A. 2 or more episodes in a single year, D. Periodic erythrocyte sedimentation rate
without radiographic clearing between (ESR)
occurrences monitoring
B. 2 or more episodes in a single year, with E. Frequent С-reactive protein (CRP)
radiographic clearing between occurrences monitoring
C. 3 or more episodes in a single year, with
radiographic clearing between occurrences 181. Antinuclear antigen (ANA)
D. 3 or more episodes in a single year, measurement test is useful in some
without radiographic clearing between rheumatologic diseases especially with
occurrences persistent
E. 4 or more episodes in a single year oligoarticular juvenile idiopathic
rheumatoid arthritis (JIA). All the following
178. Hydroxychloroquine sulfate is an are more likely to be correlated with
antimalarial drug important in the treatment ANA positivity EXCEPT
of SLE and dermatomyositis, a. Anterior uveitis
particularly cutaneous manifestations of b. Younger age at disease onset
disease and to reduce lupus flares. Of the c. Female sex
following, the MOST important d. Symmetrical arthritis
procedure that should be done routinely e. Lower number of involved joints over
during the course of administration is time
1. Gastric endoscopy
2. Bone marrow examination 182.Cyanosis in the newborn may be caused
3. Muscle biopsy by which of the following:
4. Glucose-6-phosphate dehydrogenase 1) Transposition of the great arteries.
enzyme 2) VSD
level assay 3) Hyperbilirubinaemia .
5. Ophthalmological examination 4) Coarctation of the aorta.
5) Eisenmenger syndrome
179. You are meeting parents of a 14-year-
old girl who has been treated with a monthly 183. The child with polyarticular JIA often
intravenous has a more prolonged course of active joint
cyclophosphamide for SLE-associated renal inflammation and requires
failure for the last 6 months; the mother is early and aggressive therapy. Of the
asking about the long-term following, the predictor that carries the
complications of this drug. All the following WORST prognosis is:
are long-term complications EXCEPT A. Old age at onset
A. Bone marrow suppression B. Rheumatoid factor (RF) seronegativity
B. Bladder cancer C. Absence of rheumatoid nodules
C. Leukemia D. Small numbers of affected joints
D. Lymphoma E. Hip joint involvement
E. Infertility
184. Drug-induced lupus refers to the
180. A 4-year-old girl recently diagnosed presence of SLE manifestations triggered by
with persistent oligoarticular juvenile exposure to specific medications,
idiopathic rheumatoid arthritis (JIA); including antibiotics. Of the following, the
she has 3 involved joints including the right drug that is MOST likely associated with
knee, right ankle, and left elbow; antinuclear drug-induced lupus is
antigen (ANA) is A. Isoniazid
significantly positive. Of the following, the B. Rifampin
MOST important step in the management of C. Nitrofurantoin
this girl is D. Penicillin
A. Regular examination of locomotor E. Tetracycline
system
B. Periodic slit-lamp examination
C. Periodic ANA monitoring
185. Systemic lupus erythematosus (SLE) is 3) Average systolic blood pressure (SBP)
often characterized by periods of flare and and/or diastolic BP that is ≥95th percentile
disease quiescence or may for age, sex, and height on ≥3
follow a more smoldering disease course. occasions
All the following lab tests correlate with 4) Average systolic blood pressure (SBP)
active disease EXCEPTa) high erythrocyte and/or diastolic BP that is ≥90th percentile
sedimentation rate for age, sex, and height on ≥3
b) positive anti–double-stranded DNA level occasions
c) low serum complement level 5) Average systolic blood pressure (SBP)
d) positive anti-nuclear antibody titer and/or diastolic BP that is ≥99th percentile
e) elevated C-reactive protein (CRP) value for age, sex, and height on ≥3
occasions
186. Rheumatoid factor (RF)–positive
polyarthritis is characterized by aggressive 190. What are some laboratory results you
symmetric inflammation of joints may find in a child who has acute
of both upper and lower extremities. Of the poststreptococcal glomerulonephritis?
following, the extra articular manifestation 1. A urine sample positive for glucose and
that is almost exclusively occur hematuria; a decreased blood urea nitrogen
in RF positive individuals is (BUN); and a normal
A. Fever glomerular filtration rate (GFR)
B. Evanescent rash 2. A urine sample negative for proteinuria
C. Uveitis and hematuria; a decreased blood urea
D. Extensor surfaces nodules nitrogen (BUN); a decreased
E. Pericarditis creatinine level; and a normal glomerular
filtration rate (GFR)
187. Chronic kidney disease in children <5- 3. A urine sample positive for increased
yr-old is MOST commonly a result of: sodium, potassium, and glucose; and an
A. Lupus nephritis increased glomerular filtration rate
B. Familial juvenile nephronophthisis (GFR)
C. Alport syndrome 4. A urine sample positive for proteinuria
D. Focal segmental glomerulosclerosis and hematuria; an increased creatinine level;
E. Autosomal dominant polycystic kidney and a decreased glomerular
disease filtration rate (GFR)
5. A urine sample positive for proteinuria
188. Despite the use of antibiotic agents, and hematuria; a decreased blood urea
mortality remains high, in the range of 20- nitrogen (BUN); a decreased
25%. Serious morbidity occurs creatinine level; and a normal glomerular
in 50-60% of children with documented filtration rate (GFR)
infective endocarditis. Of the following, the
MOST common morbidity is 191. What is the treatment plan for a 6-year-
A. Heart failure old boy who has acute poststreptococcal
B. Pulmonary emboli glomerulonephritis?
C. Mycotic aneurysms 1. Supportive treatment based on the
D. Acquired ventricular septal defect symptoms
E. Heart block 2. Treatment with corticosteroid
medications
189. The definition of hypertension in 3. Treatment with anti-inflammatory
children is medications
1) Average systolic blood pressure (SBP) 4. Treatment with cytotoxic medications
and/or diastolic BP that is ≥95th percentile 5. Treatment with biological agents
for age, sex, on ≥3 occasions
2) Average systolic blood pressure (SBP) 192. What are the signs and symptoms of
and/or diastolic BP that is ≥95th percentile acute poststreptococcal glomerulonephritis?
for age, sex, and height on ≥2 1. Edema, hypertension, polyuria, hematuria
occasions and proteinuria
2. Edema, hypotension, oliguria, hematuria 1. It results from a viral infection of the
and glucosuria throat or skin and tends to present 6 weeks
3. Edema, hypertension, crystalluria, after the infection.
hematuria and proteinuria 2. Clients are less likely to experience
4. Edema, hypertension, oliguria, hematuria hematuria with this condition.
and proteinuria 3. Antibodies develop and deposit within
5. Edema, hypotension, oliguria, pyuria and the kidney, and they can cause inflammation.
proteinuria 4. Clients with this condition should
consume a diet rich in sodium to prevent
193. Which of the following conditions hyponatremia.
most likely causes hematuria? 5. The body develops antibodies, which
1. Porphyria2. Tyrosinemia deposit in the ureters, causing urine to back
3. Rhabdomyolysis up into the kidney and result in
4. Post-streptococcal glomerulonephritis inflammation.
5. Hyperbilirubinemia
198. How does a streptococcal infection
194. Which of the following is associated interfere with the functioning of the kidneys?
with post-streptococcal glomerulonephritis? 1. The body develops antigens, which
1. Elevated C3, C4, and CH50 complement deposit in the nephron.
levels 2. The body develops antibodies, which
2. Low blood pressure deposit in the renal veins and cause
3. Protein-to-creatinine ratio of <0.2 inflammation.
4. Hypotension 3. The body develops antibodies, which
5. Elevated antistreptolysin O (ASO) titers deposit in the ureters, causing urine to back
up into the kidney and result in
195. Which client is most at risk of inflammation.
developing acute streptococcal 4. The body develops antibodies, which
glomerulonephritis? deposit in the glomerulus and cause
1. A 10-year-old child with the mumps inflammation.
2. A 6-year-old child with impetigo 5. The body develops antibodies, which
3. A 15-year-old girl with influenza deposit in the collecting ducts and cause
4. An 8-year-old boy with the chickenpox inflammation.
5. A 17-year-old girl with the systemic
lupus erythematosus 199. Which of the following is an indication
for imaging in urinary tract infection?
196. A 6-year-old boy was diagnosed with a 1. Failure to improve with antibiotic therapy
streptococcal infection in the throat. The 2. Acute uncomplicated urinary tract
child was sent home and began infection
taking antibiotics. During health teaching, 3. Acute pyelonephritis
the boy's mother was told to monitor the 4. Acute cystitis
child's urine output because the 5. Clinically relevant uropathogens in urine
boy was at risk of developing acute culture
streptococcal glomerulonephritis. What is
the time frame for developing this 200. Which substances tested for in the
complication of poststreptococcal throat urine dipstick test indicate a urinary tract
infection? infection?
1. 1–2 weeks 1. Leukocytes, ketones
2. 2–3 weeks 2. Protein, nitrites
3. 3–4 weeks 3. Leukocytes, glucose
4. 4–5 weeks 4. Leukocytes, nitrites
5. 6-7 weeks 5. Bilirubin, nitrites

197. Which statement about 201. Why are girls more prone to urinary
poststreptococcal glomerulonephritis is tract infections than boys?
correct?
1. Girls have longer urethras and larger 206. Which of the following best describes
bladders the relationship between acute kidney injury
2. Girls tend to urinate less frequently (AKI) and chronic kidney
3. Girls are likely to have more bacteria due disease (CKD)?
to the close proximity to the vagina4. The 1. Pre-existing CKD decreases the risk of
propensity of bacterial attachment to the AKI.
female periurethral mucosa and shorter 2. AKI and CKD are two separate, mutually
urethras may account for exclusive entities.
this difference 3. AKI can slow the progression of pre-
5. Only in girls the bacteria causing cystitis existing CKD.
ascend to the kidney to cause pyelonephritis 4. Unresolved AKI may lead to CKD.
5. AKI decreases the risk of CKD.
202. Which of the following best defines
pyelonephritis? 207. Which hormone is a part of a hormone
1. Infection of the entire urinary tract system that regulates blood pressure, fluid
2. Cystitis complicated by fever and electrolyte balance, and
3. Perirenal abscess formation systemic vascular resistance?
4. Infection of the kidneys/upper urinary 1. Angiotensin-II
tract 2. Atrial natriuretic peptide
5. Urinary tract infection with obstruction 3. Parathyroid hormone (PTH)
4. 25OH Vitamin D
203. Which of the following describes the 5. 3-Dehydroretinol
correct pathway of infection during the
pathogenesis of pyelonephritis? 208. Which of the following is an indication
1. Ureter, urethra, bladder, renal calyx, renal for kidney replacement therapy in patients
papilla with CKD?
2. Urethra, bladder, ureter, renal papilla, 1. GFR = 75 mL/min/1.73 m^2
renal calyx 2. Subjective findings of uremia (fatigue,
3. Ureter, bladder, renal calyx, ureter, renal weakness, nausea, vomiting, anorexia, and
papilla poor sleep patterns)
4. Urethra, bladder, ureter, renal calyx, renal 3. Age > 17 years
papilla 4. Serum creatinine < 1.2 mg/dL
5. Urethra, ureter, bladder, renal papilla, 5. Controlled electrolyte abnormalities
renal calyx
209. Which of the following is an indication
204. Which of the following is possible for kidney replacement therapy in patients
complication of vesicoureteral reflux? with CKD?
1. Renal scarring 1. GFR = 60 mL/min/1.73 m^2
2. Renal stones 2. Age < 1 year
3. Cystitis 3. Severe fluid restrictions that inhibit the
4. Renal vein thrombosis ability to provide appropriate nutrition
5. Hypertension sufficient for linear growth
4. Serum creatinine = 1.5 mg/dL
205. Which of the following is required for 5. Controlled electrolyte abnormalities
a diagnosis of chronic kidney disease
(CKD)? 210. Which of the following is true
1. Hematuria with dysmorphic red blood regarding the staging of CKD?1. A low
cells glomerular filtration rate measurement is
2. Heavy proteinuria (protein excretion required to diagnose CKD.
greater than 3.5 g/24 hours) 2. A patient diagnosed with stage 3 CKD
3. Oliguria (urine production less than 0.5 has very little chance of disease progression.
mL/m2/h) 3. Patients with stage 1 CKD should be
4. Multiple cysts on renal imaging treated with kidney replacement therapy.
5. Persistent evidence of kidney damage or a 4. It is intended to identify those at greatest
decrease in function for at least 3 months risk for progression and complications.
5. Earlier stages of CKD are usually 3. Creatinine production remains constant
symptomatic. among individuals with significant
variations in muscle mass.
211. What is the cause of anemia seen in 4. Serum creatinine is less affected by sex,
patients with CKD? age, and muscle mass than cystatin C.
1. Decreased red blood cell production 5. Serum creatinine is not secreted by the
2. Excessive sequestration of red blood cells renal tubules.
in the spleen
3. Vitamin deficiency 216. A patient presents with fever,
4. Deficient globin synthesis eosinophilia, elevated creatinine, and
5. Red cell membranopathy proteinuria after starting penicillin for a
Streptococcus infection and Ibuprofen
212. Which of the following is involved in because of fever. Which type of acute
the pathogenesis of mineral and bone kidney injury is most likely?
disorders seen in CKD patients? 1. Prerenal
1. Increased glomerular filtration rate (GFR) 2. Intrinsic renal
2. Inadequate activation (hydroxylation) of 3. Postrenal
vitamin D 4. Renovascular disease
3. Decreased parathyroid hormone (PTH) 5. Due to increased renal perfusion
activity
4. Increased calcitriol activity 217. A child presents with a 2-day history of
5. Decreased phosphatonin activity vomiting and diarrhea. He has oliguria and
elevated creatinine. Which
213. Which type of kidney replacement type of acute kidney injury (AKI) is most
therapy has the most favorable outcome in likely?
CKD patients? 1. Prerenal
1. Kidney transplantation 2. AKI related to acute postinfectious
2. Home hemodialysis glomerulonephritis
3. Peritoneal dialysis 3. Postrenal
4. In-center hemodialysis 4. Renovascular disease
5. Continuous hemodiafiltration 5. AKI due to increased renal perfusion

214. Which of the following is a limitation 218. A 6 yr old child with a recent
of measuring protein excretion by using a pharyngitis presents with periorbital edema,
24-hour urine collection? hypertension, gross hematuria,
1. It is difficult to perform on a regular basis. oliguria, and elevated creatinine. Which
2. It has low sensitivity to albumin. type of acute kidney injury (AKI) is most
3. It can be influenced by variations in likely?
protein production throughout the day. 1. Prerenal AKI due to decreased renal
4. It might overestimate proteinuria in perfusion
women. 2. Postrenal AKI related to anatomic
5. It can be performed only in the inpatient obstructions to the lower urinary tract
setting. 3. Intrinsic renal AKI related to acute
postinfectious glomerulonephritis
215. Which of the following explains why 4. Renovascular disease
the serum creatinine level is a useful 5. Acute tubular necrosis
measure for estimating GFR?
1. Creatinine is freely filtered across the 219. A neonate with a history of
glomerulus and is neither reabsorbed nor hydronephrosis seen on prenatal ultrasound
metabolized by the kidneys. studies presents with a palpable bladder
2. Serum creatinine can be used to estimate and elevated creatinine. Which type of acute
the GFR in individuals with unstable kidney kidney injury (AKI) is most likely?1.
function. Postrenal AKI related to congenital urinary
tract obstruction, probably posterior urethral
valves
2. Intrinsic renal AKI related to acute A. Systemic steroids
postinfectious glomerulonephritis B. Methotrexate
3. Prerenal AKI due to decreased renal C. Leflunomide
perfusion D. Cyclophosphamide
4. Renovascular disease E. Hydroxychloroquine
5. Acute tubular necrosis
225. The initial period of rickets is mostly
220. When a patient presents with true diagnosed by:
volume depletion and prerenal acute kidney A. 1 month of life
injury, which treatment is most B. 2-3 months of life
appropriate? C. 3-5 months of life
1. Diuretics D. 5-7 months of life
2. Albumin E. after a year
3. Vasopressor support
4. Intravenous fluids 226. With rickets, deformation of the lower
5. Salt and water restriction extremities occurs in:
A. 3 – 6 months
221. Which of the following is the most B. 9 – 12 months
common cause of acute tubular necrosis? C. 6 - 9 months
1. Inadequate electrolyte intake D. 1 – 3 months
2. Decreased renal perfusion E. 0 – 1 month
3. Urinary retention causing obstruction
4. Decrease in angiotensin levels 227. Common to both acute and chronic
5. Infections malnutrition is:
a) Weight for age
222. Which of the following conditions is b) Weight for height
most likely to present with acute kidney c) Height for age
injury? d) Height for weight
1) Pyelonephritis e) Height for length
2) Vesicoureteral reflux
3) Lupus nephritis 228.What gastroenterological diseases
4) Diabetic nephropathy prevalent in children?
5) Cystitis a) Isolated gastritis
b) An isolated duodenitis
223. "A 3-month-old child, against the c) Peptic ulcer.
background of subfebrile body temperature d) Duodenal ulcer
and rhinitis, has pallor, cyanosis e) Gastro duodenitis
of the nasolabial triangle, severe expiratory
dyspnea, deflated chest, dry cough, 229.The boy, 8 years old, has pale skin,
participation of auxiliary muscles in general weakness, and decreased appetite. In
breathing. and small bubbling wet rales on general blood test: Hb - 80 g/l,
both sides. In the blood: Hb - 112 g / l RBC 3,86x10 9 /l , CI 0,7, WBC - 6x10 9 /l,
1) Acute bronchiolitis bands - 2%, eosynophiles – 5%, segments -
2) Acute (simple) bronchitis 51%, lymph. - 32%, monocytes
3) Bilateral pneumonia -10%, thromb. - 210х10 9/l ESR - 7
4) Bronchial asthma, exacerbation mm/hour. What disease is possible in tis
5) Acute obstructive bronchitis case?
a) Iron-deficiency anemia
224. A 12-year-old female adolescent has a b) Hemolytic anemia
recent diagnosis of SLE; she has a faint c) Acute leukemia
malar rash and mild arthritis d) B12-deficiency anemia
including both elbow and knee joints as well e) Aplastic anemia
as the joints of small fingers. Of the
following, the MOST appropriate
initial therapy for this patient is
230. Have 4-month girl child complains of c) Ascorbic acid 30 mg 3 times a day.
anxiety, sleep fitfully, poor appetite. The d) Calcium gluconate 400-500 mg per day.
baby was born premature from a e) Vitamin D 1000 IU per day.
second pregnancy weight at the time of birth
was 2400 g, the skin and visible mucous 234. Define the characteristic feature of
membranes pale. Heart rate - subacute rickets.
144 in 1 min. Heart sounds loud, rhythmic a) Kraniotabes
activity, tachycardia, mild systolic murmur. b) Prevalence of osteomalacia
The liver serves 4 cm from the c) Prevalence of osteoid hyperplasia
edge of costal arch. Analysis of blood: red d) The presence of severe hypocalcemia
blood cells - 2.5 * 10 12 degree / l, e) Rickety rosary
hemoglobin - 88 g / l, color index -
0.72, reticulocytes - 0.7%. What kind of 235. Define the calcium content in serum at
anemia should think? hypervitaminosis D.
a) Iron deficiency anemia severity, a) Calcium levels above 2.7 mmol / l
Hypochromic b) Within the physiological oscillations.
b) Iron deficiency anemia severity, c) Calcium levels lower than 2.2 mmol / l
hyperchromic. d) Calcium levels upper than 1.8 mmol / l
c) Hypoplastic anemia. e) Calcium levels less than 1.8 mmol / l
d) Iron deficiency anemia and severity
e) Aplastic anemia 236. The first level of diet therapy at
hypotrophy is called:
231. The patient, 15 years old, 5 years a) Rest and minimum diet
suffering from duodenal ulcer. After b) Increase in the caloric input (intermediary)
exercise felt severe pain in the epigastric c) Returning to normal regime
region and right upper quadrant. It was d) Primary diet
vomiting. There is a weak pulse, sharp pain e) Secondary diet
from the navel to the right, the
disappearance of hepatic dullness during 237. The earliest sign of the development of
percussion. What caused condition that vitamin d-deficient rickets is
requires emergency care? a) Deformity of the lower extremities
a) Duodenal ulcer perforation b) Delayed physical development
b) Spontaneous pneumothorax c) Deformation of the skull bones
c) Aggravation of kidney stones (renal colic) d) Vegetative disorders
d) Exacerbations of cholelithiasis e) Mental retardation
e) Pyloroduodenal stenosis
238. A pediatrician examines a healthy full-
232. Hypotrophy (CND) degree III (Severe term baby at the age of 1 month, who is on
malnutrition) characterized by: natural feeding. What kind of
a) Deficit weight of standards for10 – 20% disease prevention will the doctor
b) Deficit weight of standards for 20 – 30% recommend first?
c) Deficit weight of standards more than 30% a) Iron deficiency anemia
d) Deficit weight of standards 30 – 60% b) Hypervitaminosis D
e) Deficit weight of standards 30 – 50% c) Acute respiratory viral infection
d) Rickets
233. The child, 2 months, was born in e) Dystrophy
October, weighing 3000 g, is on the
breastfed. Now the weight is 4400 grams, 239. The child was hospitalized for 11
neuro-psychological development age. months due to ricketogenic tetany. After the
During the inspection, clean skin, pink; emergency treatment, the condition
organs and systems - no change. What stabilized, the seizures did not recur.
is the prevention of rickets be assigned to Determine further therapeutic tactics.
the child? 1.
a) Vitamin D 500 IU per day. Prescribe large doses of vitamin D
b) Vitamin D to 2000 IU per day. 2.
Prescribe large doses of calcium ESR – 8 mm / hour. Which conclusion is
3. correct?
Prescribe glucocorticoids 1. Anemia.
4. 2. Epilepsy
Prescribe anticonvulsant therapy 3. Spasmophilia.
5. 4. Phosphate is diabetes
Prescribe calcium supplements and anti- 5. Rickets, acute period
rickets 6. Iron deficiency anemia
treatment
243. In a child of 5 months. against the
240. The child is 1.5 months old. He was background of rickets treatment (oral
born prematurely with a weight of 2000g. vitamin D3 in a dose of 5 thousand IU
During pregnancy, the mother and total Ultraviolet irradiation) there was
suffered from gestosis of the 1st and 2nd an attack of tonic-clonic seizures. Which
half of pregnancy. For 2 weeks, there is examination should be
increased sweating, excitability. prescribed first?
What is the most likely pathological 1.
condition? The consequences of perinatal Determination of the level of calcium and
damage to the nervous system.1. phosphorus in the blood.
Rickets. 2.
2. Determination of the level of calcium and
Functional disorder of the gastrointestinal phosphorus in the urine
tract. 3.
3. Spinal tap
Spasmophilia. 4. Neurosonography
4. 5.
The initial manifestations of acute Computed tomography of the brain
respiratory
viral infection 244. A 3-month-old child was diagnosed
5. with rickets. Which acid plays an important
Iron deficiency anemia role in the metabolism of
phosphorus and calcium, and affects the
241. The pediatrician, after examining 2- action of calciferol, the function of the
month-old Masha, determined rickets, first parathyroid glands?
degree of severity, initial 1. Acetic acid
period, acute course. What is the daily dose 2.
of an aqueous solution of vitamin D3 Salt
needed for treatment? 3.
1. Phosphoric
2000 IU (4 drops) 4.
2. Sulfuric acid
3000 IU (6 drops) 5.
3. Lemon
4000 IU (8 drops)
4. 245. The parents of a 6-month-old child are
5000 IU (10 drops) concerned about the rapid growth of his
5. head, since the hats bought earlier
6000 IU (12 drops) turned out to be small. The measurements
carried out by the nurse showed that the
242. The mother of 3-month-old Sasha circumference of the child's head
complains of restless sleep in the child, is 43 cm, the chest is 45 cm. Evaluate these
decreased appetite. On examination: parameters.
flattening of the occiput, softening of the 1.
edges of the large fontanel, hypocalcaemia Proportional and appropriate to the age of
in the blood test, Hb – 110 g / l, the
child 4.
2. Rickets
Both parameters have been increased 5.
3. Spasmophilia
Increased head size
4. 249 A month-old child has anxiety,
The chest is enlarged increased sweating of the head. From the
5. anamnesis of life: since his birthday
Both parameters are below the age limit (September 5), he has been fed cow's milk.
Upon examination, craniotabes is noted.
246. A child with manifestations of rickets The doctor prescribed a course
deficiency developed laryngospasm and of UV radiation. Decide on the need to
carpopedal spasm after prescribe vitamin D3 preparations to this
prolonged exposure to the spring sun. This child.
is typical for: 1.
1. 2 – 2.5 months after the end of the course of
Febrile seizures Ultraviolet irradiation2.
2. There is no need
Encephalitis 3.
3. Together with the Ultraviolet irradiation
Meningitis 4.
4. Immediately after completing the course of
Hypoparothyroiditis the Ufa
5. 5.
Ricketogenic tetany 1 month after the end of the Ultraviolet
irradiation course
247. Hereditary rickets-like diseases include
1. Vitamin D-dependent rickets, vitamin D- 250. A 3-month-old child has baldness of
resistant rickets, de Toni-Debre-Fanconi the back of the head, restless sleep,
disease, galactosemia increased sweating. What kind of
2. Vitamin D-dependent rickets, vitamin D- pathological condition can you think of?
resistant rickets, de Toni-Debre-Fanconi 1. Anemia
disease, renal tubular acidosis 2.
3. Vitamin D-dependent rickets, vitamin D- Spasmophilia
resistant rickets, cystic fibrosis, renal 3.
tubular acidosis Rickets
4. Nonspecific ulcerative colitis, vitamin D- 4.
resistant rickets, de Toni-Debre-Fanconi Phosphate is diabetes
disease, renal tubular acidosis 5.
5. Vitamin D-dependent rickets, renal Chondrodystrophy
tubular acidosis, vitamin D-resistant rickets,
de Toni-Debre-Fanconi disease 251. The boy is 9 months old. during the
scream, noisy breathing appeared, cyanosis
248. A pediatrician examined a two-month- of the skin, cold sweat, short
old child. Mother's complaints of periodic term apnea, tonic cramps in the arms and
anxiety, excessive sweating. legs. After a few minutes, the child became
On examination: the back of the head is active again. Upon examination,
flattened, bald. The edges of the large only signs of rickets were revealed, body
fontanelle are pliable. What kind of temperature – 36.6 ° C. It is fed with breast
disease are we talking about? milk. Which drug should be
1. Vitamin D-resistant rickets prescribed first after an attack.
2. 1. Vitamin C
Phosphate diabetes 2. Vitamin D
3. 3.
Debre-de Toni-Fanconi syndrome Calcium Gluconate
4. 3.
Sodium Oxybutyrate Calcipenic Rickets
5. 4.
Finlepsinum Rickets of Prematurity
5. Vitamin D Resistant Rickets
252. In a 3-year-old child, when applying to
a preschool institution, changes in the bone 254. A 2-year-old boy with severe
system were revealed: impairments in physical and psychomotor
thickenings on the ribs at the junction of the development and skeletal deformities is
bone and cartilaginous parts - "rosaries", suspected to have Debre-de Toni-Fanconi
deformation of the sternum, X syndrome. What changes in urine confirm
shaped deformities of the lower extremities. this diagnosis?
The child was staying with his grandmother 1.
until he was 3 years old, and Galactosuria, lactosuria
was not observed by the family doctor. 2.
Make a plan for follow-up and rehabilitation Glucosuria, hyperaminoaciduria
measures. 3. Acetonuria, isostenuria
1. 4.
Prednisone with a short course of 5-7 days, Proteinuria, hematuria
1-2 mg / kg of body weight per day, exclude 5.
foods rich in calcium, Cylindruria, bacteriuria
vitamin E, C, group B. Prevention: strict
observance of the rules, instructions for the 255. The child was 6 months old. against the
use of the dosage form of background of a mild course of Acute
vitamin D. respiratory infection, repeated
2. Control of physical and neuropsychiatric clonic seizures appeared. Upon examination,
development, in case of deviation - exercise signs of rickets of moderate severity were
therapy, massage, calcium diagnosed. Blood calcium is
D3 Nicomed 1.2 mmol/l. There is no data on perinatal
3. Introduction of calcium–enriched foods damage to the central nervous system. The
into the diet cerebrospinal fluid parameters
4. Immediate hospitalization, after are normal. Artificial feeding. What is the
clarification of the diagnosis – dispensary most likely condition?
registration, monitoring of 1. Neurotoxicosis
development and implementation of 2.
recommendations of specialists Meningitis
5. 3.
Control of physical and neuropsychiatric Encephalitis
development, in case of deviation - exercise 4.
therapy, massage, calcium Encephalitic reaction
D3 Nicomed, daily stay in the fresh air, the 5.
introduction of calcium–enriched foods into Spasmophilia
the diet
256. 1.8-year-old boy was brought to the
253. Given the following data, determine doctor by his mother complaining of
the clinical course of rickets characterized frequent abdominal pain and diarrhea
by hypertrophy of the frontal and after consuming dairy products.
parietal eminences, thickening of the Examination revealed reduced levels of
interphalangeal joints of the fingers: skeletal lactase in the intestines. Which of the
deformities, growth retardation, following conditions is most likely in the
bone pain and tenderness, dental anomalies, child?
muscle weakness, muscle cramps, seizures: 1.
1. Nutritional Rickets Selective IgA deficiency
2. 2.
Hypophosphatemic Rickets Cow's milk allergy
3. syndrome
Polycystic ovary syndrome 5.
4. Undifferentiated immunoglobulin Irritable Bowel Syndrome (IBS)
enteropathy
5. 260. Children with сhronic
Lactase deficiency malnutritionshould discontinue therapeutic
feeding only when they meet the following
257.A 5-year-old girl named Anya was criteria:
brought to the emergency department by her 1. Weight/height or weight/length will be at
mother with complaints of chronic least ≥ –2 SD
abdominal pain, diarrhea, and poor growth. 2.
Upon examination, she was found to exhibit Mid-upper arm circumference will be < 125
fatigue and irritability, as mm
well as nutrient deficiencies including iron 3. Absence of edema for 2 days
and vitamin D. Serological tests revealed 4.
elevated levels of anti-tissue Mid-upper arm circumference will be < 50
transglutaminase antibodies (anti-tTG) and mm
anti-endomysial antibodies. A small bowel 5. Weight/height or weight/length will be at
biopsy confirmed the presence least ≥ –4 SD
of villous atrophy. Which of the following
conditions is most likely in the child? 261. The maximum diagnostic program for
1. detecting hypotrophy in malabsorption
Selective IgA deficiency syndrome includes:
2. 1.
Cow's milk allergy Comprehensive blood tests
3. 2.
C.Polycystic ovary syndrome Stool pH determination
4. Undifferentiated immunoglobulin 3.
enteropathy Evaluation of fecal analysis
5. 4.
Celiac disease Collection and assessment of the child's
biological and genealogical history
258.Causes of prenatal hypotrophy: 5. Analysis of pancreatic enzyme levels in
1. stool (lipase, amylase).
Ectopic pregnancy
2. 262. Which form of prenatal dystrophy can
Hypoplasia of the pancreas be considered inconsistent with the other
3. listed forms?
Phenylalanine intolerance 1. Neurotic
4. 2. Neuroendocrine
Chromosomal abnormalities 3. Neurodystrophic
5. 4.
Maternal malnutrition Encephalopathic
5. Neurovegetative
259.Which diseases can lead to secondary
lactose intolerance due to damage to the 263. Hypostature is -
mucous membrane of the small 1.
intestine? Excess body weight in relation to height
1. 2. Uneven lag in body weight and height
Celiac disease from age
2. Crohn's disease norms
3. 3.
Rotavirus infection Deficiency of body weight relative to height
4. 4. Uniform lag in body weight and height
Gut-associated lymphoid tissue (GALT) from age
norms 266. For the 4-year-old child: At birth, the
5. Uniform lag in body weight relative to child weighed 3500 g, with a body length of
height 50 cm, head circumference
of 35 cm, and chest circumference of 32 cm.
264. 4-year-old Maxim was brought to the The mother consulted a doctor because the
doctor by his mother with complaints of child is not gaining weight
edema, frequent abdominal pain, well. What weight should the 4-year-old
and diarrhea. Upon examination, signs of child have?a) Weight 20 kg.
edema were detected, including abdominal b) Weight 15 kg.
and lower extremity edema. The c) Weight 16 kg.
child also appears apathetic and fatigued. d) Weight 16.5 kg.
Objective examination revealed a low level e) Weight 18 kg.
of albumin in the blood and the
absence of facial edema. Laboratory 266.For the 5-month-old child: The child is
analyses showed a low level of proteins and 5 months old and is exclusively breastfed.
hypoalbuminemia. Based on the During a check-up, the doctor
symptoms and laboratory data, Maxim is diagnosed the child with 1st degree
suspected of having kwashiorkor, a form of hypotrophy. What is the most desirable first
protein-energy malnutrition. complementary food for this child?
Which of the following treatment directions a) Vegetable puree
is most suitable for 4-year-old Maxim b) 5% semolina porridge
suspected of kwashiorkor? c)
1. 10% semolina porridge
Initiate feeding through a gastrostomy tube d) Buckwheat porridge
to ensure an adequate intake of nutrients e) Oatmeal porridge
2.
Recommend liver transplantation to correct 267.Child is 4 months old, with a birth
metabolic disorders weight of 3350 g. Two months ago, the
3. child experienced an acute intestinal
Conduct urgent gastroenterological infection, after which unstable stool and
examination to detect possible regurgitation appeared. At the time of
complications examination, the body weight is 5000
4. g, with decreased tissue turgor, thinning of
Recommend gastroenterological subcutaneous fat layer on the torso and
rehabilitation and long-term treatment in limbs. Muscular hypotonia,
stationary conditions to stabilize the hepatomegaly are observed. Preliminary
condition diagnosis: 2nd degree hypotrophy. What
5. volume of feeding is necessary for
Prescribe intensive pharmacological therapy this child?
for rapid elimination of edema and a)
normalization of digestion 1/4 of the age-appropriate amount
b) According to age
265. Infant is 2 months old, born at term c)
with a weight of 3500 g. Breastfeeding is 1/5 of the age-appropriate amount
partial. Currently weighs 4900 g. d)
Evaluate the infant's body mass. 1/3 of the age-appropriate amount
a) According to age e)
b) 1/2 of the age-appropriate amount
150 g below normal
c) 268.A 4-month-old child has been
1st degree hypotrophy diagnosed with 2nd degree hypotrophy.
d) 2nd degree hypotrophy What is the deficit in body weight for the
e) child?
3rd degree hypotrophy a)
21-30%
b) e)
10-20% 800 ml
c)
31-50% 272.A full-term newborn boy was born with
d) 5-10% a body weight of 3800 g, and on the 3rd day
e) 51-70% of life, his body weight is
3200 g. Evaluate the weight loss in the child.
269.Child is 30 days old, born prematurely a) Doesn't matter
with a body weight of 2300 g. The mother b) Cannot be assessed
has hypogalactia, and the child c) Below the norm
gained 300 g in the first month. What d) Within the norm
assumption would be optimal for the family e) Exceeds the norm
doctor in this situation?
a) 273.A 4-month-old child has been
Supplementation with donor milk experiencing frequent watery stools from
b) Supplementation with 5% semolina the first days of life. Breastfeeding is
porridge natural. After consuming milk, the mother
c) Transition to artificial feeding with cow's experiences abdominal pain and liquid stool.
milk The child is active, but has a
d) Transition to artificial feeding with 24% deficit in body weight. Stool occurs 3-
adapted 5 times a day, watery and acidic-smelling.
formulas Examination revealed: sweat
e) Observation over time chloride 20.4 mEq/L, stool culture -
negative. What is the probable cause of
270.A 1.5-year-old child, born with a body hypotrophy in the child?
weight of 3100 g and a body length of 51 a) Enteritis
cm, is breastfed. After introducing b) Congenital lactose deficiency
complementary feeding (oatmeal porridge), c) Celiac disease
the child stopped gaining weight, and there d) Cystic fibrosis
were feces with a strong e) Exudative enteropathy
unpleasant odor in large quantities.
Objectively, the child exhibits pale skin, an 274.The child sleeps restlessly, gnashes his
enlarged abdomen, and signs of 3rd teeth in his sleep, combs the perianal area.
degree hypotrophy. What deficit in body Thin white lines with a length
weight corresponds to this degree of of 1 cm and pointed ends were revealed.
hypotrophy? Find out the diagnosis?a) Trichocephalosis
a) b) Ascariasis
5-10% c) Trichinosis
b) d) Enterobiosis
11-20% e) Opisthorchiasis
c) 21-30%
d) 31% and above 275. A 2-month-old girl is on artificial
e) Doesn't matter feeding. How often should the dynamics of
body weight be monitored in a
271.A 2-month-old girl is being transitioned satisfactory condition of the child to prevent
to artificial feeding. She was born with a hypotrophy?
body weight of 3900 g. a) Twice a week
Currently, her body weight is 3900 g. b) Once every 2 weeks
Specify the daily feeding volume for this c) Once every 3 weeks
child. d) Once a month
a) e) Twice a day
650 ml 276.A family doctor reviews the basic
b) 730 ml anthropometric data of two girls, aged 13
c) 750 ml and 15, from the same family. The
d) 600 ml
body mass index (BMI) is most commonly were found in the feces. Tell the most
used for assessment. It is: appropriate diagnosis?
a) Body weight multiplied by height a) Ascariasis
b) Body weight divided by height b) Hookworm
c) Height divided by body weight c) Dracunculosis
d) Height squared minus body weight d) Loaosis
e) Body weight (kg) divided by height e) Trichinosis
squared (m)
281.Helminth larvae were accidentally
277.A mother consulted a family doctor discovered in the laboratory tests during
regarding her 2-week-old baby because he is sputum microscopy
gaining weight poorly. Prenatal of a pneumonia patient. During the blood
hypotrophy of the II degree was diagnosed test, eosinophilia was detected. What kind
in the full-term baby, characterized by the of helminthosis can be
following weight-to-height ratio: expected?
a) a) Vuhereriosis
80 b) Ascariasis
b) 75-80 c) Trichocephalosis
c) 70-75 d) Paragonimiasis
d) 60-70 e) Opisthorchiasis
e)
50-55 282.During puncture of a liver cyst (tumor
with fluid), small nodular formations in the
278.A 6-month-old child with a birth weight form of grains of sand were
of 3000 g, whose physical development revealed in a transparent, barely yellowish
corresponds to his age, is on liquid. What kind of helminthiasis can be
artificial feeding with adaptive formulas. foreseen?
Determine the daily food volume for the a) Echinococcosis
child. b) Fascioliasis
a) c)
500 ml Schistosomiasis
b) 600 ml d) Hymenolepiasis
c) 700 ml e) Cysticercosis
d) 900 ml
e) 283.Large oval yellowish eggs with a dark
1000 ml brown uneven shell were found in the feces
of a patient with a digestive
279.The child is 5 months old, with a birth disorder, a dark mass in the middle, and free
weight of 3000 grams, currently weighing spaces in the shape of a crescent at the poles.
5500 grams. They are partially What is the diagnosis?
breastfed (receiving 500 ml of breast milk a) Teniasis
and 200 ml of "Baby" formula per day). The b) Ascariasis
appetite is good, and there c) Fascioliasis
are no digestive disorders. What is the cause d) Trichinosis
of hypotrophy in this case? e) Trichocephaliasis
a) Mother's hypogalactia
b) Syndrome of impaired intestinal 284.The patient has headache, muscle pain
absorption during movement, swallowing, chewing and
c) Intestinal infection eye rotation, weakness,
d) Dysbacteriosis fever, swelling of the eyelids and face.
e) Alimentary factor There are no eggs in the feces and perianal
280. In a patient with severe anemia and area. What is the probable
allergic phenomena, small mobile worms of helminthiasis?
red color with a size of 1 cm a) Cysticercosis
b) Trichinosisc) Hookworm
d) Echinococcosis confirmed during surgery. From which
e) Trichocephalosis animal could the patient have contracted
echinococcosis?
285.Echinococcus was detected in a) Pigs
laboratory rabbits during autopsy. A rabbit, b) Cows
like a human, is an additional host for c) A rabbit
the parasite: d) Dogs
a) carrier e) Cats
b) intermediate host
c) 290.During the operation, small bubbles of
final host small size with an insignificant amount of
d) reservoir host liquid were detected in the
e) none of the above patient's liver, which fit tightly one to the
other. What kind of helminthiasis was
286.A sick child periodically has abdominal revealed in the patient?
pain, loose stools, and nausea. One day, a a) Fasciolosis
cylindrical white worm of 15 b) Alveococcosis
cm was released with vomiting. What kind c) Opisthorchiasis
of laboratory test should be carried out? d) Echinococcosis
a) Detection of segments in the feces e) Dicroceliosis
b) Detection of helminth larvae in the
muscles by biopsy 291.Treatment of the patient with
c) Ovoscopy of the perianal zone by pneumonia did not significantly alleviate his
scraping or using adhesive tape condition. He began to complain of
d) Examination of feces and duodenal pain in his stomach, nausea and other
contents on eggs digestive disorders, deterioration of his
e) Fecal examination for eggs general condition. A laboratory
analysis of feces intended for doctors
287. White helminths of 5-10 mm were revealed the presence of oval-shaped
found in the feces, with a vesicular helminth eggs covered with a thick
expansion of the esophagus in front of bumpy shell. What is the diagnosis?
them. The eggs were found not in feces, but a) Fasciolosis
in scraping from the perianal folds, colorless, b) Trichocephalosis
non-symmetrical, oval. c) Enterobiosis
What is the diagnosis? d) Ascariasis
a) Hookworm disease e) Diphyllobothriosis
b) Enterobiosis
c) Teniosis 292. Two children with pinworms have
d) Trichinosis been identified in kindergarten. What kind
e) Trichocephalosis of preventive measure should be
carried out to prevent infection of other
288.For the prevention of which children?
helminthiasis is it necessary to follow the a) Cook meat and fish well
rules of personal hygiene? b) No need for any activities
a) Fascioliasis c) Wash fruits and vegetables well
b) Diphyllobothriosis d) Make vaccinations
c) Teniarinchiasis e) Disinfect toys
d) Opisthorchiasis
e) Echinococcosis 293.Name which of the helminthiasis listed
below may be the cause of chronic
289.A doctor put a preliminary diagnosis to appendicitis:
an employee of a livestock farm: a) ascariasis, enterobiosis, trichocephalosis
echinococcosis. The diagnosis was b) trichinosis, hookworm, paragonimosis
c) vuhereriosis, trichinosis, hookworm
d) brugiosis, loaosis, opisthorchiasis
e) teniosis, trichocephalosis, fasciolosis d) Pinworm
e) Dwarf chain
294.Which nematode larvae migrate
through the human bloodstream during the 299. Primary hypovitaminosis occurs:
development cycle? a) as a result of insufficient intake of
a) Whipworms, intestinal eels, filaria vitamin(s) into the body or due to imbalance
b) Ascaris, pinworms, crooked c) of nutrition
Hookworms, trichinella, ascaris b) as a result of partial destruction of
d) Headed pinworms, necator, ascaris vitamins in the digestive tract
e) Whipworms, hookworms, intestinal eels c) as a result of increased intake of proteins,
fats and carbohydrates into the body
295.During microscopy of a smear of feces d) as a result of increased destruction of
of a schoolboy, yellow-brown eggs with a proteins, fats and carbohydrates in the body
bumpy shell were revealed. e) as a result of increased intake of proteins
Which helminthiasis is it? and fats into the body
a) Pinworm
b) Human Ascaris 300.Water-soluble vitamins are include:
c) Dwarf a) vitamins A and D
d) Chainworm – Human Whipworm b) vitamins E and K
e) Broad Lent c) vitamins PP
D) vitamins B6
296.A 10-year-old child complains of e) vitamins C
weakness, nausea, irritability. White
helminths were found on length 5-10 301. Find the characteristic symptoms of
mm. During microscopy of the scraping hypovitaminosis A.
from the perianal folds, colorless eggs in the a) Athe growth and development of the
form of asymmetric ova were body slows
revealed. What kind of helminth parasitizes down
a child? b) Body weight increases;
a) Human Ascaris c) Hypokeratosis.
b) Duodenum Crooked head d) Delayed physical development
c) Pinworm e) Mental retardation
d) Trichinella
e) Whipworm 302.Define the most characteristic age for
hypovitaminosis D.
297.Echinococcosis is one of the most a) young children;
dangerous human helminthiases requiring b) conscripts;
surgical intervention. What method c) teenagers 16-18 years old
is used for laboratory diagnosis of this d) newborn children
disease? e) preschool children
a) X–ray
b) Ovohelmintoscopy 303.Thiamine deficiency occurs when:
c) Immunological a) with a diet high in protein
d) Larvogelmintoscopy b) when eating a diet with a high proportion
e) Biological samples of
refined carbohydrates
298.During microscopy of the scraping from c) sleep disorders and depression
the perianal folds, colorless eggs were d) decrease in blood pressure
revealed in the child, having the e) when eating with a decrease in protein
shape of asymmetric ovals, measuring content
50×23 microns. What kind of helminthiasis
is? 304. Symptoms of folic acid
a) Ascaris hypovitaminosis occur when:
b) Duodenum a) the diet contains folic acid at a level of 50
c) Human Whipworm mcg/day;
b) less than 5 mcg/day of folic acid enters examination of the child, signs of
the body with food for a long time; exhaustion, dry skin, and brittle hair were
c) it is impossible to determine the amount found. Laboratory tests showed a low
of folic acid entering the body. level of iron in the blood, as well as low
d) lack of awareness of the population about levels of vitamins D and B12. Which of the
the need to prevent folic acid deficiency following methods is the most
effective for diagnosing hypovitaminosis in
305. Hypovitaminosis E is considered a risk 6-year-old Denis?
factor for: a) Abdominal ultrasound
a) Pathologies of pregnancy b) Computerized tomography (CT) of the
b) Diseases of the genitourinary system of brain
boys c) Urine analysis for protein content
c) Atherosclerosis and angina pectoris d) Measurement of vitamin levels in the
d) Damage to the skin blood
e) Decrease in blood pressure306. The toxic e) Determination of growth and weight for
effect of calciferol (vitamin D) usually age using specialized growth charts.
occurs when:
a) long-term increase in the diet of special 309.8-year-old Andrei was brought to the
vitamin preparations containing vitamin D pediatrician by his parents with complaints
in increased physiological of frequent episodes of fatigue,
doses reduced activity, and increased irritability.
b) the use in the diet of adulterated The parents also note that Andrei often
vegetable oil intended for animal feed and experiences headaches and unusual
artificially enriched with vitamin D changes in his usual behavior. Objective
c) diseases caused by vitamin D deficiency examination revealed signs of muscular
against the background of compensatory weakness, paleness of the skin, and
therapy increased nail fragility. Laboratory tests
d) diseases caused by vitamin C deficiency showed a low level of iron in the blood, as
against the background of compensatory well as a low level of vitamin
therapy C.Which of the following methods is the
e) use of products containing ascorbic acid most effective for diagnosing vitamin C
in the diet deficiency in 8-year-old Andrei?
a) Prescribing oral vitamin supplements
307. The toxic effect of vitamin C usually containing vitamin C
occurs when: b) Intravenous administration of vitamin C
a) long-term increase in the diet of special preparations
vitamin preparations containing vitamin C c) Recommending increased consumption of
b) use of products containing ascorbic acid fruits and vegetables rich in vitamin C
in the diet d) Applying skin patches with vitamin C
c) diseases caused by vitamin C deficiency e)Using homeopathic remedies to increase
against the background of compensatory the level of vitamin C in the body
therapy
d) use of adulterated vegetable products in 310.7-year-old Anna was brought to the
the diet pediatrician by her parents with complaints
e) use of foods containing vitamins in the of increased fatigue, sleep
diet disturbances, and irritability. Parents also
note that Anna frequently experiences
308.6-year-old Denis was brought to the muscle and bone pain, especially
pediatrician by his parents with complaints during prolonged activity. Upon objective
of weakness, rapid fatigue, and examination, signs of muscle weakness and
poor appetite. The parents also note that increased bone sensitivity were
Denis often experiences sleep disturbances identified. Laboratory tests revealed low
and mood changes. Upon objective levels of calcium and phosphorus in the
blood, as well as a low level of
vitamin D.Which of the following treatment c) Vitamin D 4000-10000 IU per day
approaches is most suitable for 7-year-old d) Vitamin D 2000-4000 IU per day
Anna suspected of vitamin D e) Vitamin D 1000-2000 IU per day
deficiency?
a) Prescribing oral vitamin complexes 314. The mother complains of lack of teeth
containing vitamin D, with a focus on and lower limb deformation in a child 8
calcium and magnesium. months. The baby was born
b Intravenous administration of vitamin D weighing 3000 grams healthy from birth on
preparations with regular monitoring of artificial feeding, prevention of rickets
blood calcium levels. received. During the inspection,
c) Recommending increased consumption of the overall condition is not violated, pale
fatty fish such as salmon and cod, rich in skin, teeth no large fontanel has a size of 3 x
vitamin D. 3 cm lower aperture chest
d) Using specialized capsules with expanded palpable ribs rosary. There O-
micronutrients and vitamins, including curvature of the lower extremities. With the
vitamin D, to raise their levels in the weakening of the heart listen
body. colors, functional systolic murmur. What is
e) Conducting a course of extravagant your diagnosis?
therapy using vitamin injections, including a) Rickets
vitamin D, in combination with b) congenital brittle bones
homeopathic remedies to boost immunity. c) Hondrodystrofy
d) vitamin D-dependent rickets
311. A pediatrician examined a two-month- e) Vitamin D-resistant rickets
old child. Mother's complaints of periodic
anxiety, excessive sweating. 315. A child 6.5 months, was born weighing
On examination: the back of the head is 3500 g, body length 54 cm. Located
flattened, bald. The edges of the large breastfed. Mother complains of poor
fontanel are pliable. What kind of appetite, anxiety child, pale skin and visible
disease are we talking about? mucous membranes, recurrent abdominal
1. Vitamin D-resistant rickets cramps, liquid stool. Disturbing
2. Phosphate diabetes symptoms listed above during the last 2
3. Debre-de Toni-Fanconi syndrome weeks against the backdrop of the
4. Rickets introduction of the second breast feeding
5. Spasmophilia in the form of porridge. The examination:
heart rate - 136 in 1 min. Heart sounds loud,
312. What is the most important sign of tachycardia, mild systolic
B12-deficiency anemia: murmur over the top. Belly swollen, hard on
a) Hyperchromic anemia palpation. The liver acts at 3 cm from the
b) Hypochromic anemia edge of costal arch. In the
c) Thrombocytopenia analysis of blood: red blood cells - 2.8 * 10
d) Anemia, lymphopenia, monocytopenia 12 degree / l, hemoglobin - 80 g / l, color
e) Anemia, reticulocytosis index - 0.76. What was the cause
of anemia?
313. The child, 3 months, child gets cow's a) Malabsorption syndrome
milk. Mother complains of anxiety, sleep b) Diseases of the digestive tract
disturbances, increased sweating. inflammatory
During the inspection: general condition c) Lack of iron in food
was violated, attracts attention pallor, hair d) Breastfeeding
loss back, it is flat, asymmetrical, e) Protein-energy deficiency
a slight softening of the skull bones, the ribs
are the rosary in a small amount. Organs 316. An 8-month-old infant presents with
and systems - within the age the primary complaint of irritability. He has
appropriate. What treatment should be? been exclusively breastfed since
a) Vitamin D in 400-600 IU per day
b) calcium in the age dose
birth. His mother was not interested in 1.
providing any supplemental foods because Colonoscopy
her milk supply has been adequate. 2.
Physical examination reveals a fussy infant H2 inhibitors
who has frontal bossing and whose weight 3.
and height are both at the 25th Continue proton pump inhibitors
percentile. The infant becomes irritable with 4.
movement of the left arm. Arm radiography Endoscopy with biopsy
reveals a humeral fracture 5. Abdominal ultrasonography320.What is
and bowing of both radii. Chest radiography an effective test for detection of H. pylori
demonstrates enlargement of the infection?
costochondral junctions. 1. Ultrasound scan
of the following, the most likely diagnosis is 2. Urea breath
a) Osteogenesisim perfecta 3.
b) Vitamin D-deficient rickets Blood picture
c) Vitamin D-resistant rickets 4. Urine analysis
d) Vitamin E deficiency 5.
e) Hondrodystrofy glomerular filtration rate (GFR)

317. Pneumonia during auscultation is 321.Which of the following medical


characterized by: conditions is the most common cause of
a) The presence of dry wheezing, which dyspepsia?
increases on exhalation, wheezing breathing 1.
b) The presence of crepitation or small- Functional dyspepsia
bubbly wet wheezing, weakened or 2.
bronchial breathing Gastric cancer
c) The presence of dry wheezing, scattered 3.
throughout the pulmonary fields, hard Peptic ulcer disease
breathing 4.
d) The presence of small bubbly wheezes at Pancreatitis
the height of inspiration, various wet and 5.
dry wheezes, which change the Esophageal cancer
character after coughing
e) The presence of wet and dry wheezing 322. Which structure of Helicobacter pylori
318. A 7-year-old child present with allows it to attach to the superficial gastric
hypoalbuminemia, edema,hyperlipidemia cells?
and proteinuria.The edema 1. Cell wall
is 2. Cell membrane
in the periorbital region initially and 3. Cilia
eventually spreads 4. Flagella
to the rest of the body. The patient is given 5. Pseudopodia
steroid therapy and the disease goes 323. Which of the following is the best next
away.What is a key morphological feature step in the management of Functional
of the patient’s disease? dyspepsia with alarm features?
a. Fusion of the foot processes a) Proton pump inhibitors
b. Destruction of the basement membrane b) Testing for Helicobacter pylori
c. Destruction of the glomerulus c) Endoscopy
d. Hemosiderin laden macrophages in the d) Diet and lifestyle modification
kidney e) H2 antagonists
e. None of the above 324. Which of the following
pathophysiological mechanisms is NOT
319.Which of the following is the best next responsible for injury in peptic ulcer disease?
step after 8 weeks of proton pump inhibitor 1.
trial without improvement of Reflux of acid into the esophageal mucosa
chronic epigastric discomfort? 2.
Mucosal damage due to Helicobacter pylori Blocks hydrochloric acid production by the
and NSAIDs gastric glands
3.
Loss of blood supply to the mucosa 328. A 17-year-old girls presents to the
4. emergency department with complaints of
Increased acid production pain in her abdomen that started
5. an hour after eating lunch. Which of the
Inhibition of prostaglandins following is the diagnostic modality of
choice in this patient?
325. What types of medication are 1.
considered for a client with peptic ulcer CT scan with contrast
disease (PUD)? 2.
a) H2 receptor antagonists, antibiotics, CT scan without contrast
proton pump inhibitors, probiotics 3. Ultrasound
b) H2 receptor antagonists, antibiotics, 4. Nuclear medicine scan
proton pump inhibitors, mucosal protectants 5. X-ray
c) H2 receptor antagonists, antibiotics,
probiotics, antacids 329. An obese teenager comes to the ER
d) H2 receptor antagonists, nonsteroidal with episodes of vomiting and RUQ pain
anti-inflammatory drugs, proton pump radiating to her right scapula.
inhibitors, mucosal protectants Which of the following imaging modality
e) Nonsteroidal anti-inflammatory drugs, would be most helpful to confirm acute
antibiotics, proton pump inhibitors, mucosal cholecystitis?
protectants, probiotics 1.
CT abdomen
326. Which of the following is a side effect 2. Abdominal ultrasound
of omeprazole? 3.
1. MRI abdomen
Increased risk of sepsis 4.
2. Plain x-ray of abdomen
Increased risk of developing hirsutism 5.
3. Esophagogastroduodenoscopy
Increased risk of enteric infection
4. 330. A 17-year-old girl presents with right
Increased risk of anxiety upper quadrant pain. She has a BMI of 32
5. kg/m2. She complains of
Increased risk of developing dermatitis feeling nauseated at times. What is the
nigricans probable type of gallbladder stone causing
the right upper quadrant pain?
327. What is the mechanism of action of a) Pure cholesterol stoneb) Mixed stones
ranitidine? c) Combined stones
1. d) Pigment stone
Blocks histamine receptor type 2 in parietal e) Uric acid stone
cells
2. 331. What kind of supports can help infants
Blocks histamine receptor type 2 in the manage viral bronchiolitis?
chief cells a.
3. Antipyretics, nasal suction, hydration,
Blocks histamine receptor type 1 in the supplemental oxygen
surface mucous cells b. Antibiotics, nasal suction, hydration,
4. stopping breastfeeding
Blocks hydrochloric acid production by the c.
G cells Antipyretics, nasal suction, fluid restriction,
5. supplemental oxygen
d. Antibiotics, nasal suction, hydration, positive blood culture
supplemental oxygen
e. 335. Which of the following is a common
Antipyretics, stopping breastfeeding, cause of acute pneumonia, especially in the
supplemental oxygen pediatric population?
a) Mycoplasma pneumonia
332. When a patient is diagnosed with acute b) Nocardia
bronchitis, which of the following c) Actinomycosis
investigations is usually performed d) Tuberculosis
in order to rule out an accompanying e) Aspergillus
pneumonia?
1. Chest X-ray 336. What age range of children should be
2. Bronchoscopy treated with amoxicillin for bacterial
3. HRCT-scan pneumonia?
4. Lung biopsy a.
5. MRI Children 5–8 years old
b.
333. 16-year-old male presents with a Children < 5 years old
history of productive cough and low grade c.
fever. He was previously healthy Children 5–10 years old
and does not suffer from any chronic d.
ailments. He is diagnosed with Children > 5 years old
tracheobronchitis. What is the next step in e.
management? Adolescents
a)
Supportive care only 337. In patients with health care-associated
b) Inhaled steroids pneumonia, which cephalosporin is initially
c) Inhaled bronchodilators included in the antibiotic
d) Oral third generation cephalosporins coverage regimen?
e) Oral neuraminidase inhibitors 1. Ceftriaxone
2. Cefuroxime
334. Which factors would indicate a 3. Cefepime
diagnosis of bronchitis? 4. Cefaclor
a) History of acute onset of persistent cough 5. Cefazolin
for 1–3 weeks, no clinical signs of
pneumonia (fever, rales, 338.Renal childhood diseases responsible
tachypnea), an infiltrate on chest radiograph for chronic hypertension includea)
b) History of acute onset of persistent cough hemolytic-uremic syndrome
for 1–3 weeks, no clinical signs of b) acute tubular necrosis
pneumonia (fever, rales, c) congenital dysplastic kidney
tachypnea), chest X-ray to rule out d) pyelonephritis
pneumonia e) renal trauma
c) History of acute onset of persistent cough
for 1–3 weeks, no clinical signs of 339.Mother of a previously healthy 4 year
pneumonia (fever, rales, old male complains of cough and wheeze.
tachypnea), a pleural effusion or empyema Boy had playing with a small
on chest radiograph toy. During examination the right side of a
d) History of acute onset of persistent cough chest show hyperresonance, diminished
for 1–3 weeks, no clinical signs of vocal resonance and poor air entry.
pneumonia (fever, rales, What is the most probable diagnosis?
tachypnea), an elevated WBC count, in the a. Pneumonia
range of 15,000-40,000/mm3 b. Asthma
e) History of acute onset of persistent cough c. Foreign body aspiration
for 1–3 weeks, clinical signs of pneumonia d. Bronchitis
(fever, rales, tachypnea), e. Bronchiolitis
340.Boy is 3 years old. Complaints:
dyspnoe, fast tiredness, frequent episodes of
respiratory diseases in history.
Borders of relative heart dullness are
extended to the left, strengthening of the
2nd heart sound in the 2nd intercostal
interspace on the left side, hard systolo-
dyastolic murmur in the second intercostal
interspace on the left side and
above the clavicle (“machine noise”), which
is conducted on interscapular interspace.
What is the most probable
diagnosis?
a. Aortal stenosis
b. Patent arterial duct
c. Atrial septal defect
d. Ventricle septal defect
e. Isolated stenosis of arteria pulmonalis

You might also like